You are on page 1of 58

Chapter – 1

PERIODIC CLASSIFICATION OF ELEMENTS


AND CHEMICAL BONDING
1.1 Introduction
It is very difficult to study individually the chemistry of all the elements and millions of their
compounds, hence to simplify and systematize the study of chemistry of the elements and their
compounds, they are classified into groups and periods.

Dobereiner’s law of traids: It was first attempt towards classification. He arranged similar
elements in a group of three elements called triad and the atomic mass of the middle elements
of the traid is approximately the arithmetic mean of the other two.

e.g., Ca40, Sr87.5, Ba137.

137  40
At. wt. of Sr =  88.5
2

88.5 is nearly similar to 87.5 of atomic wt. of Sr.

Newland’s law of octaves: When the lighter elements are arranged in order of their increasing
atomic weights, then every eighth element is similar to the first element in its properties, similarly
as the eighth note of a musical scale is similar to 1st one, e.g., Na 8th element resembles in their
properties with Li. Similarly K the 8th element with Na and so on.

Lother Meyer arrangement: According to him the elements which are similar in their chemistry
occupied similar positions on the graph of atomic volume vs atomic weights, e.g., (a). The alkali
metals occupy the peaks on the curve while alkaline earth metals occupy the descending position
in the curve. (b) Halogens occupy the ascending positions on the curve.

Mendeleev’s periodic law: Initially he proposed that the chemical properties of the elements
are the periodic function of their atomic weights.

But later on, he modified the statement after considering the Lother Meyer’s work. Now, the
combined statement is called Mendeleev’s periodic law “The physical and chemical properties of
the elements are the periodic function of their atomic weights”.

Refer to Mendeleev’s table on next page.

Periodicity: The repetition of similar properties after a certain fixed interval.

Periodic table: The table which classifies all the known elements in accordance with their
properties in such a way that the elements with similar properties are grouped together in the same
group while dissimilar elements are in the other group.

[1]
Periodic Classification of Elements and Chemical Bonding

[2]
Periodic Classification of Elements and Chemical Bonding

Merits of Mendeleev’s periodic table:


(a) Study of elements and their compounds becomes easy and systematic, as by knowing the
property of one element in a group, then the properties of the other elements present in the
same group can easily be predicted.
(b) Helps in the discovery of new elements. As Mendeleev’s left some blank spaces for some
unknown elements and further, predicted the properties of these elements, e.g., ekaluminium,
ekasilicon.
(c) Correction of doubtful atomic masses.
(d) Correction in the valency of some element.
(e) Correction in the position of some elements.
Limitation of Mendeleev’s periodic table:
(i) Position of hydrogen is anomalous as it resembles with 1st group alkali metal and also with
7th group halogen in their properties.
(ii) Position of isotopes: Isotopes must have different positions but they are placed in the same
group.
(iii) Position of isobars : Are placed in different groups.
(iv) Dissimilar elements are placed together in the same group like K & Cu in 1st group.
(v) Similar elements placed in different groups.
(vi) Some higher atomic weight elements placed before the lower atomic weight elements, e.g.,
Ar40 precedes K39, Co58.9 precedes Ni58.7, Te127.6 precedes l127.
(vii)Position of metals and non-metals: Both are placed together in the same group.
(viii) Diagonal relationship could not be explained.
(ix) Position of lanthanides and actinides are not properly specified.
(x) No proper position to VIII group elements.
(xi) Position of noble elements.
(xii) Existence of variable valency could not be explained.
(xiii) Cause of periodicity can not be explained.
Main features of Mendeleev’s Periodic Table:
(i) Elements are arranged in order of their increasing atomic weights.
(ii) It consists nine vertical columns called groups, designated as I to VIII and zero. (Zero group
was added later on).
(iii) It consists of seven horizontal rows called periods 1 to 7.

(iv) VIII group has three elements in a row (groups).

(v) Zero group contains inert gases.

[3]
Periodic Classification of Elements and Chemical Bonding

[4]
Periodic Classification of Elements and Chemical Bonding

Modern periodic law : The physical and chemical properties of the elements are the periodic
function of their atomic number.

Cause of periodicity: It is due to the repetition of similar outer shell electronic configuration at
a certain regular intervals.

Structural features of the long form of the periodic table:

(i) It consists of 18 vertical columns called groups and 7 horizontal columns (or rows) called
periods.

(ii) Elements of groups 1, 2, 13 – 17 are called normal or representative elements.

(iii) Elements of groups 3 – 12 are called transition elements.

(iv) The 14 elements with atomic numbers (Z) = 58 – 71 (occurring after lanthanum 57La in the
periodic table) are called lanthanides or rare earth elements and are placed at the bottom
of the periodic table. The 14 elements with atomic numbers (Z) = 90 – 103 (Occurring after
actinium 89Ac in the periodic table) are called actinides and are placed at the bottom of the
periodic table.

(v) The Eleven elements with Z = 93 – 103 (93Np – 103Lr) which occur in the periodic table after
uranium and have been prepared from it by artificial means are called transuranics. These
are all radioactive elements.

(vi) The elements belonging to a particular group are said to constitute a chemical family which
is usually named after the name of the first element. For example, Boron family (group 13),
carbon family (group 14), nitrogen family (group 15), and oxygen family (group 16). In addition
to this, some groups have typical names. For example,

Elements of group 1 are called alkali metals.

Elements of group 2 are called alkaline earth metals.

Elements of group 16 are called chalcogens.

Elements of group 17 are called halogens.

Elements of group 18 are called zero group or noble gases.

The long form of the periodic table contains seven periods. These are :

1st period (1H – 2He) contains only two elements. This is the shortest period.

2nd period (3Li – 10Ne) and third period (11Na – 18 Ar) contain 8 elements each and are
called short periods.

4th period (19K – 36 Kr) and 5th period (37Rb – 54Xe) contain 18 elements each and are
called long periods.

6th period (55Cs – 86Rn) contains 32 elements and is the longest period.

7th period (87Li –) is, however, incomplete and contains at present only 24 elements.

[5]
Periodic Classification of Elements and Chemical Bonding

In yet another classification, the long form of the periodic table has been divided into four
blocks (i.e., s, p, d and f ), depending upon the subshell to which the last electron enters.

s-block elements: Elements of groups 1 and 2 including He in which the last electron enters
the s-orbitals of the valence shell are called s-block elements. There are only 14 s-block
elements in the periodic table.

p-block elements: Elements of groups 13–18 in which the last electron enters the p-orbitals
of the valence shell are called p-block elements.

d-block elements: There are three complete series and one incomplete series of d-block
elements. These are: 1st or 3d-transition series which contains ten elements which atomic
numbers 21–30 (21Sc – 30Zn).

2nd or 4d-transition series which contains ten elements with atomic numbers 39 – 48 (39Y – 48Cd).

3rd or 5d transition series which also contains ten elements which atomic numbers 57 and
72 – 80 (57La, 72Hf – 80Hg).

4th or 6d transition series which is incomplete at present and contains only nine elements.
These are 89Ac, 104Rf, 105Ha, Unh (Unnihexium, Z = 106), 107Ns (Neilsobohrium), 108Hs
(Hassium), 109Mt (Meitherium), Uun (Ununnilium, Z = 110) and Uud (Unundium, Z = 112) or
Ekamercury. The element, Z = 111 has not been discovered so far. Thus, in all there are 39
d-block elements.

(vii) f-Block elements are also called inner-transition elements. In these elements, the
f-subshell of the anti-penultimate is being progressively filled up. There are two series of f-
block elements each containing 14 elements. The fourteen elements from 58Ce – 71Lu in
which 4 f-subshell is being progressively filled up are called lanthanides or rare elements.
Similarly, the fourteen elements from 90Th – 103Lr in which 5 f-subshell is being progressively
filled up are called actinides.

Illustration 1: Which of the following elements belongs to inner transition elements?


(a) Tc (b) Dy
(c) Cs (d) Fr

Solution: (b) Dysprosium (atomic number 66).

Illustration 2: Eka mercury is the name given to the element having atomic number
(a) 112 (b) 86
(c) 110 (d) 104

Solution: (a)

SAMPLE PROBLEMS 1.1 (MCQ)


Problem 1: Alkaline earth metals belong to
(a) 1st group (b) 2nd group
(c) 3rd group (d) 4th group

Solution: (b)

[6]
Periodic Classification of Elements and Chemical Bonding

Problem 2: Which of the following sets of elements are commonly known as halogens?
(a) Na, K, Rb, Cs (b) F, Cl, Br, I
(c) O, S, Se, Te (d) N, P, As, Sb
Solution: (b)
Problem 3: Which of the following sets of elements are commonly known as Pnicogens?
(a) Na, K, Rb, Cs (b) F, Cl, Br, I
(c) O, S, Se, Te (d) N, P, As, Sb
Solution: (d)
Problem 4: Which of the following sets of elements are commonly known as Chalcogens?
(a) Na, K, Rb, Cs (b) F, Cl, Br, I
(c) O, S, Se, Te (d) N, P, As, Sb
Solution: (c)
Problem 5: Which of the following is not an actinide?
(a) Curium (b) Californium
(c) Uranium (d) Terbium
Solution: (d)
Problem 6: Europium is
(a) s-block (b) p-block
(c) d-block (d) f-block
Solution: (d)
Problem 7: Element with atomic number 56 belongs to which block?
(a) s (b) p
(c) d (d) f
Solution: (a)
Problem 8: Which of the following sets is coinage metals?
(a) Cu, Ag, Au (b) Zn, Cd, Hg
(c) Au, Ag, Zn (d) Li, Na, K
Solution: (a)
Problem 9: The electronic configuration of transition elements is exhibited by:
(a) ns1 (b) ns2.np5
(c) ns2. (n – 1)d10 (d) (n – 1)d1 – 10ns0 – 2
Solution: (d)
Problem 10: An element X belongs to fourth period and fifteenth group of the periodic table. Which
one of the following is true regarding the outer electrons configuration of X? It has
(a) partially filled d-orbitals and completely filled s-orbital
(b) completely filled s-orbital and completely filled p-orbitals
(c) completely filled s-orbital and half filled p-orbitals
(d) half filled d-orbital and completely half filled p-orbitals

Solution: (c)

[7]
Periodic Classification of Elements and Chemical Bonding

1.2 Diagonal relationship:


Certain elements of 2nd period show similarity with their diagonal elements in the 3rd period as
shown below:
Group 1 Group 2 Group 13 Group 14
2nd period Li Be B C
3rd period Na Mg Al Si P

Thus, Li resembles Mg, Be resembles Al and B resembles Si. This is called diagonal relationship
and is due to the reason that these pairs of elements have almost identical ionic radii and polarizing
power (i.e., charge/size ratio). Elements of seconds period are known as bridge elements.

Anomalous behaviour of the first element of a group: The first element of a group differs
considerably from its congeners (i.e., the rest of the elements of its group). This is due to (i) small
size (ii) high electronegativity and (iii) non availability of d-orbitals for bonding. Anomalous
behaviour is observed among the second row elements (i.e., Li to F).

Illustration 1: Diagonal relationship of elements of second period to those of third period is due to
(a) same polarising power (b) same ionic radii
(c) same electronegativity (d) absence of vacant d-orbitals

Solution: (a) polarising power  charge / size

Periodic Properties: Properties which are directly or indirectly related to their electronic
configuration and show a regular gradation when we move from left to right in a period or from
top to bottom in a group are called periodic properties. Some important periodic properties are
atomic size, ionization energy electron affinity, electronegativity, valency, density, atomic volume,
melting and boiling points, etc.

(a) Atomic size: It refers to the distance between the centre of the nucleus of the atom to the
outermost shell containing electrons. Since absolute value of the atomic size cannot be
determined, it is usually expressed in terms of the following operational definitions.
X X

A B

1/2 AB = rcovalent
(of element X)

X H H X X
H
E F
G H

1/2 EF = rvan der Waals of hydrogen in HX molecule


1/2 GH = rvan der Waals of X in HX molecule
n 2a 0
rn 
Z*
[8]
Periodic Classification of Elements and Chemical Bonding

(i) Covalent radius: It is defined as one-half of the distance between the nuclei of two
covalently bonded atoms of the same element in a molecule.

(ii) van der Waals’ radius: It is defined as one-half of the distance between the nuclei of
two non-bonded isolated atoms or two adjacent atoms belonging to two neigbhouring
molecules of an element in the solid state.

By definition, van der Waals’ radius of an element is always larger than its covalent radius.

Variation of atomic radii: (i) Across the period atomic radii decreases.

(ii) Where, we move from 17th group to 18th atomic radii increases
the period decreases.

(iii) The shrink in atomic radii across the period decreases.

Variation in Period :

Li Be B C N O F Ne

Z 3 4 5 6 7 8 9 10

Z* 1.30 1.95 2.60 3.25 3.90 4.55 5.20 5.85

n 2 2 2 2 2 2 2 2

rn (pm) 123 90 80 77 75 74 72 160

(covalent) (van der Waals)

(b) Ionic size: An atom can be changed to a cation by loss of electrons and to an anion by gain
of electrons. A cation is always smaller than the parent atom because during its formation
effective nuclear charge increases and sometimes a shell may also decrease. On the other
hand, the size of an anion is always larger than the parent atom because during its formation
effective nuclear charge decreases, e.g.

Mg2+ < Mg , Cl– > Cl.

(c) Isoelectronic ions or species are the neutral atoms, cations or anions of different elements
which have the same number of electrons but different nuclear charge e.g.,

The size of the isoelectronic species depends upon their nuclear charge Greater the nuclear
charge, lesser the radii.

In the main groups, radii increase on descending the group because extra shell is added

Li+ Na + K+
76 pm 102 pm 138 pm

The ionic radii decrease moving from left to right across any period in the periodic table

Na + Mg2+ Al3+
102 pm 72 pm 53.5 pm

[9]
Periodic Classification of Elements and Chemical Bonding

Ionization Energy (IE) It is the amount of energy required to remove the most loosely bound
electron from an isolated gaseous atom, i.e.

M (g) + IE  M+ (g) + e–

The amount of energies required to remove the first, second, third, etc., electrons from the isolated
gaseous atom are called successive ionization energies and are designated as IE1`, IE2, IE3,
etc. It may be noted that IE 2 is always greater than IE 1 and IE3 is greater than IE 2, i.e.,
IE3 > IE2 > IE1.

The factors which affect the ionization energies are :

Factors affecting Ionisation Energy (I.E.):

(i) Nuclear charge, (ii) Atomic size, (iii) Penetration effect of the electrons, (iv) Screening effect
of the inner electrons and (v) Effect of exactly half filled and completely filled orbitals.

(a) If nuclear charge increases ionization energy increases.

(b) Ionization energy varies inversely with atomic radii, ionization energy increases with decrease
in atomic radii.

(c) The nature of sub shell from where electron will be knock out it is also known as penetrating
effect. I.E. of different sub shell are in the order of s > p > d > f.

(d) Screening or shielding effect decreases ionization energy.

(e) Orbital configuration of electrons : The I.E. of different orbitals are in the order

completely filled orbitals > completely half filled.orbitals > partly filled orbitals.

Electron Affinity (EA) It is the amount of energy released when a neutral isolated gaseous atom
accepts an electron to form gaseous anion.

X (g) + e  X– (g) + EA

Similarly, second and third electron can be added to form gaseous dinegative and trinegative ions.
The energy changes accompanying the addition of first, second, third, etc., electrons to neutral
isolated gaseous atoms are called successive electron affinities and are designated as EA1, EA2,
EA3, etc.

Since an atom has a natural tendency to accept an electron, therefore, the first electron affinity
(EA1) is always taken as positive. However, the addition of second electron to the negatively
charged ion is opposed by coulombic repulsion, hence, energy has to be supplied for the addition
of second electron. Thus, second electron affinity (EA2) of an element is taken as negative. For
example,

O (g) + e–  O– (g); EA1 = + 141kJ mol–1  HEA1 = –141 kJ/mol


– – 2– –1
O (g) + e  O (g); EA2 = – 780 kJ mol HEA2 = +780 kJ/mol

Factors affecting Electron affinity:

(i) Size of atom :- When the size of the atom is large electron affinity is small.

(ii) Nuclear Charge :- Greater the nuclear charge higher the electron affinity.
[ 10 ]
Periodic Classification of Elements and Chemical Bonding

(iii) Electronic configuration:- Elements having completely filled or half filled orbitals do not
accept the electron easily so their electron affinity are either very small or almost zero.

Trend in electron affinity (i) Cl > F > Br > I (ii) F > O > N

Electronegativity (EN), It is the tendency of an atom in a molecule to attract the shared pair
of electrons towards itself. It depends upon (ii) atomic size and (ii) nuclear charge.

Applications of electronegativity.

(i) It helps to predict the polarity of bonds and dipole moment of molecules. If the EN difference
between the two atoms is more than 1.7, the bond is considered to be ionic since it has more
than 50% ionic character. If, however, the difference is less than 1.7, it is considered to be
covalent.

(ii) It helps to have an idea about the atomic size. Higher the electronegativity, smaller is the
atomic size.

(iii) Similarly, it helps to have an idea about the bond length. Higher the electronegatively
difference, smaller is the bond length.

Mulliken method,

IE  E.A
E.N. = in e.V..
2
IE  EA
E.N. = in kJ/mol.
540
IE  EA
E.N. = in Kcal/mol.
130

where IE and EA are in electron volts.

Pauling scale  A   B  0.108  . where,  = (BE of A – B) – (sum of geometric means of


BE of A – A and B–B).

Factor affecting electronegativity: Period


(a) Electronegativity increases with Atomic radii Decreases
decrease in atomic size of the
IE Increases
element.
Groups

EA Increases
(b) Greater the I.E. of E.A. higher
would be the electronegativity. Electronegativity Increases
Decreases
Increases

(c) Elements or species having higher


positive charge would have higher
electronegativity.

(d) Anions have lower electronegativity.

(e) Greater the oxidation state higher the electronegativity.

[ 11 ]
Periodic Classification of Elements and Chemical Bonding

Metallic character: It is the tendency to lose electrons.

On moving from left to right in a period metallic character decreases but increases on moving
top to bottom in a group. e.g., Metallic character across the period, Na > Mg > Al

Metallic character down the group e.g., group - I Cs > Rb > K > Na > Li

Non-metallic character: It is the tendency to accept the electron.

On moving from left to right in a period the non-metallic character increases due to increase in
the effective nuclear charge but decreases on moving top to bottom in a group as on increase
in the number of shell force of attraction decreases.

Reactivity: In general, the reactivity of the element decreases from top to bottom in a group
except IA and IIA. In these groups it increases. While in the period on moving left to right no
such decrease or increase are observed.

In case of metals generally the reactivity increases with decrease in ionization energy, so on the
moving left to right in a period I.E., increases, then the reactivity of metal decreases. In case
of non-metals the reactivity is proportional Electronegativity & electron affinity so it increases in
a period from left to right but decreases in a group from top to bottom.

Oxidation state: Except the IA and IIA group elements all the other elements shows variable
oxidation state. IA & IIA group elements show +1 and +2 state respectively.

Valency : IA & IIA group element shows +1 and +2 oxidation state. While most of the other
group element shows variable valency. The VIIA group elements shows –1 electrovalency and
one co-valency. The variable co-valency of p-block elements from top to bottom down the group
varies as,

(i) Covalent state change in to electrovalent.

(ii) Stability of higher valent state decrease while that of lower valent state increases.

Acid-base behaviour of oxides and hydroxides: The oxide or hydroxide of an element may
act either a base or an acid depending upon its ionization energy. If the ionization energy is low,
it acts as a base and if the ionization energy is high, it acts as an acid. The ionization energies
of alkali metals are the lowest, therefore, their oxides and hydroxides are the strongest bases.
Further, since the ionization energies of alkali metals decrease down the group, the basic character
of their hydroxides increases in the same order: CsOH > RbOH> KOH> NaOH> LiOH.

The ionization energies of halogens are quite high, therefore, their oxides are the strongest acids.
Further, since the ionization energies decrease down the group, therefore, the acidic character of
their oxides and hydroxides decreases in the same order: HClO4 > HBrO4 > HlO4

As we move from left to right within a period, the ionization energies usually increase and hence
their oxides and hydroxides show a gradual variation from strongly basic through
amphoteric to strongly acidic character. For example,

Na2O MgO Al2O3 P4O10 SO2 Cl2O7


Strongly basic Basic Weakly acidic Weakly acidic Very strongly acidic
acidic

[ 12 ]
Periodic Classification of Elements and Chemical Bonding

Hence, Cs2O > Rb2O > K2O > Na2O > Li2O Basic nature (Down the group).

Ba(OH)2 > Sr (OH)2 > Ca(OH)2 > Mg(OH)2 Basic nature of hydroxide.

Atomic Volume: Atomic volume may be defined as the volume occupied by one mole atoms of
the element at its melting point in solid state. It is obtained by dividing the gram atomic mass of
the element by its density.

Atomic volume decreases along the period, reaches a minimum in the middle and then starts
increasing. This is due to the different packing arrangement of atoms in different elements in the
solid state. For example, P4, S8, etc.

In moving down the group atomic volume goes on increasing gradually.

Illustration 2: Which one of the following statements is incorrect in relation to ionisation enthalpy?
(a) Ionisation enthalpy increase for each successive electron.
(b) The greater increase in ionisation enthalpy is experienced on removal of electron from
core noble gas configuration.
(c) End of valence electrons is marked by a big jump in ionisation enthalpy.
(d) Removal of electron from orbitals bearing lower a value is easier than from an orbital
having higher n value.

Solution: (d) Removal of electron from orbitals bearing lower value of n is difficult than from the
orbital having higher value of n.

Illustration 3: Which transition involves maximum amount of energy?


(a) M  (g) 
 M(g)  e (b) M  (g) 
 M  (g)  2e

(c) M  (g) 
 M 2 (g)  e (d) M 2 (g) 
 M 3 (g)  e

Solution: (d) 3rd I.P. > 2nd I.P. > 1st I.P.

Illustration 4: The electronic configurations for some neutral atoms are given below:
(A) 1s2, 2s2 2p6, 3s2 (B) 1s2, 2s22p6, 3s1
(C) 1s2, 2s22p6, 3s2, 3p2 (D) 1s2, 2s22p6, 3s23p3
Which of these is expected to have the highest second ionisation enthalpy?
(a) (A) (b) (B)
(c) (C) (d) (D)

Solution: (b) B atom after losing outermost electron acquires noble gas configuration (stable
configuration). It is difficult to remove the next electron from B+ (1s2, 2s22p6) ion.

SAMPLE PROBLEMS 1.2 (MCQ)


Problem 1: Which of the following has lowest ionisation energy?
(a) Oxygen (b) Nitrogen
(c) Fluorine (d) Sulphur

Solution: (d) Sulphur is a member of 3rd period while O, N and F are members of 2nd period. The
atomic radius of sulphur is higher than O, N and F.
[ 13 ]
Periodic Classification of Elements and Chemical Bonding

Problem 2: Identify the correct order of the size of the following:


(a) Ca2+ < K+ < Ar < Cl– < S2– (B) Ar < Ca2+ < K+ < Cl– < S2–
(c) Ca2+ < Ar < K+ < Cl– < S2– (d) Ca2+ < K+ < Ar < S2– < Cl–

Solution: (a) In isoelectronic species, the radii decreases as atomic number increases.

Problem 3: The electronic configurations of four elements are given below. Arrange these elements
in the correct order of the magnitude (without sign) of their electron affinity.
(i) 2s22p5 (ii) 3s22p5
(iii) 2s22p4 (iv) 3s23p4
Select the correct answer using the codes given below:
(a) (i) < (ii) < (iii) < (iv) (b) (ii) < (i) < (iv) < (iii)
(c) (i) < (iii) < (iv) < (ii) (d) (iii) < (iv) < (i) < (ii)

Solution: (d) The element (ii) is oxygen and (iv) is sulphur. The electron affinity of sulphur is higher
than oxygen
The element (i) is fluorine and (ii) is chlorine. The electron affinity of chlorine is more
than fluorine. The electron affinity of fluorine is more than oxygen. Thus, the order
is (iii) > (iv) > (i) < (ii)

Problem 4: The electronic configuration of the atom having maximum difference in first and second
ionisation energies is:
(a) 1s2, 2s22p6, 3s2 (b) 1s2, 2s22p6, 3s2
(c) 1s2, 2s22p1 (d) 1s2, 2s22p6, 3s23p3

Solution: (a) The element (a) is an alkali metal. It has the maximum difference in first and second
ionisation energies is after loss of one electron, alkali metal atom acquire inert gas
(stable) configuration.

Problem 5: Which one of the following arrangements represents the correct order of electron gain
enthalpy (with negative sign) of the given atomic species?
(a) F < Cl < O < S (b) S < O < Cl < F
(c) O < S < F < Cl (d) Cl < F < S < O

Solution: (c) Consult problem 3.

Problem 6: The correct order of first ionisation energy among the following elements
Be, B, C, N, O is
(a) B < Be < C < O < N (b) B < Be < C < N < O
(c) Be < B < C < N < O (d) Be < B < C < O < N

Solution: (a) Be and N having stable configuration have abnormal values of IE1.

Problem 7: Which one of the following is an incorrect statements?


(a) The ionisation enthalpy of nitrogen is greater than that of oxygen.
(b) The electron affinity of fluorine is greater than that of chlorine.
(c) The ionisation potential of beryllium is greater than the of boron.
(d) The electronegativity of fluorine is greater than that of chlorine.

Solution: (b) The electron affinity of fluorine is less than that of chlorine.

[ 14 ]
Periodic Classification of Elements and Chemical Bonding

Problem 8: Among the element with following electronic configurations, which one of them may have
the highest ionisation energy?
(a) [Ne] 3s23p3 (b) [Ne] 3s23p2
(c) [Ar] 3d104s24p3 (d) [Ne] 3s23p1

Solution: (a) Has stable configuration and small radius.

Problem 9: The correct order of electronegativity of halogen is:


(a) Cl > Br > I > F (b) Br > I > Cl > F
(c) I > Br > Cl > F (d) F > Cl > Br > I

Solution: (d) Electronegativity decreases in a group.

Problem 10: The electronegativity of the following elements increases in the order:
(a) C, N, Si, P (b) N, Si, C, P
(c) Si, P, C, N (d) P, Si, N, C

Solution: (c) Electronegativity decreases in a group and increases in a period.

Chemical Bonding
1.3 Introduction
Why do atoms combine? It was observed that atoms of noble gases had little tendency to
combine with one another or with atoms of other elements because of the greater stability of their
ns2 np6 valence shell electronic configuration. Therefore, it was concluded that atoms of other
elements combine with one another to acquire stable noble gas configuration. This tendency of
atoms to acquire a maximum of 8 electrons in the valence shell is called octet rule. However, this
rule is not universally applicable since stable molecules are known in which the number of
electrons in the valence shell is either less (e.g., BF3, AlCl3, etc.) or more (PCl5, SF6, etc.) than
eight electrons. All such stable molecules which contain less 8 electrons in the valence shell are
called electron-deficient while those which contain more than 8 electrons in the valence shell
are called hypervalent compounds. Later on, it was found, that even noble, gases combine to
form compounds such as XeF2, KrF6, XeOF2, XeO3, etc. Thus, the octet rule failed.

Modern Theory: According to modern theory of chemical bonding, atoms form bonds since it
leads to decrease in energy. In fact whenever two atoms come close, both attractive and repulsive
forces operate. If the magnitude of the attractive forces is more than those of repulsive forces,
the potential energy of the system decreases and a chemical bond is formed. For example, two
hydrogen atoms combine to form H2 molecule. In contrast, two helium atoms do not combine to
form helium molecule since here repulsive forces dominate over the attractive forces.

Types of bonds: Some important types of chemical bonds are : (a) ionic (b) covalent (c)
co-ordinate and (d) metallic. In addition, attractive interactions between atoms of different
molecules give weak bonds of the following types:

(i) Hydrogen bond and (ii) van der waal’s interactions.

[ 15 ]
Periodic Classification of Elements and Chemical Bonding

Ionic or Electrovalent bond is formed by complete transfer of one or more electrons from the
valence shell of one atom to the valence shell of the other. The atom that losses electrons gets
converted into a cation while the other which gains electrons get converted into an anion.
·· –
· ·· :  Na+ [: Cl
Na + · Cl
.. :] or Na+ Cl–
··
Favourable conditions for forming stable ionic compound:

(i) Low ionization energy (IE) of the metal.

(ii) High electron affinity (EA) of the non-metal.

(iii) High lattice energy (U) of the resulting ionic compound. For lattice energy to be high, the force
of electrostatic attraction between the constituent ions should be high. The force of attraction
between two oppositely charged ions in air (vacuum) is given by Coulomb’s law.

q1q2 q1q2
F   F  4  r 2 0 = permeability constant
r2 0

where q1 and q2 are the respective charges on the ions and r is the distance between them
which is obtained by adding the radii of the two ions.

The lattice energy depends upon:

(a) Type of crystal lattice

(b) Force of attraction between cation and anions hence, higher charge density on cations and
anion gives rise to greater lattice energy.

Order of lattice energy of certain compounds are:

(i) LiX > NaX > KX > RbX > CsX


(ii) MgO > CaO > SrO > BaO
(iii) MgCO3 > CaCO3 > SrCO3 > BaCO3
(iv) MgSO4 < CaSO4 < SrSO4 < BaSO4
(v) Mg(OH)2 > Ca(OH)2 > Sr(OH)2 > Ba(OH)2

As lattice energy increases, melting and boiling points of ionic compounds increase. This is
due to strong electrostatic force of attraction.

Characteristics of ionic compounds:

(i) Due to strong electrostatic attraction between the ions, ionic compounds have high melting and
boiling points

(ii) In the solid, the ions are strongly attracted by one another and hence are not free to move.
Therefore, in the solid state, ionic compounds do not conduct electricity. However, in fused
(molten) state or in aqueous solution, the ions are free to move and hence conduct electricity.

(iii) In accordance with the general principle of “Like dissolves like”, ionic compounds being highly
polar are soluble in polar solvents like H2O but are insoluble in non-polar solvents like benzene,
CCl4, CS2, etc.

[ 16 ]
Periodic Classification of Elements and Chemical Bonding

(iv) Since electrostatic forces of attraction act in all directions, therefore, ionic compounds do not
possess directional characteristics and hence do not show stereoisomerism.

(v) In aqueous solutions, ionic compounds from free ions which readily undergo ionic reactions.
For example,

Na+ Cl– (aq) + Ag+ NO3– (aq)  AgCl (s) + Na+ NO3– (aq)

Covalent Bonding:

(a) Covalent bond is formed by mutual sharing of electrons so as to complete their octets
or duplets in case of H, Li and Be. Depending upon whether one, two and three electrons
are shared by each atom, single, double and triple bonds are respectively formed. The
number of electrons contributed by each atom for sharing is called its covalency.

(b) Valence-Bond Theory-Orbital Concept. According to this theory, a covalent bond is


formed between two atoms by partial overlap of their half-filled atomic orbitals containing
electrons with opposite spin.

(c) Sigma () and pi ()-bonds. When two half-filled atomic orbitals overlap along their
internuclear axis, the two p-orbitals, one s- and p-orbital. Accordingly these bonds are
designated as s-s, p-p and s-p sigma bonds.

On the other hand, covalent bond formed by sideways or lateral overlap of p-orbitals is
called pi-bond (or -bond). It may be noted that:

(i) All single bonds are -bonds.

(ii) Multiple bonds contain one -bond and the rest are -bonds.

(iii) A -bond is never formed alone. First a -bond is formed and then the formation of
the -bond takes place.

Sigma () and Pi () Bonds:

The bonds formed by end to end overlapping of orbitals are called  bonds. Now, since the two
orbitals overlap along their axis maximum overlapping is possible and hence the bond formed ()
is strong. It is formed by the overlapping of two s orbitals or one s and one p or the two p orbitals
of the two different atoms. The -bonds however, are formed by lateral overlapping of atomic
orbitals. -bonds are weaker than -bonds.
-Bond

 

s-p Overlapping Axial p-p Overlapping


s-s Overlapping

[ 17 ]
Periodic Classification of Elements and Chemical Bonding

Differences Between  – and - Bonds

Sl. No. Sigma () Bond Pi () Bond

1. If results from the end to end It results from the sideways (lateral)
overlapping of two s orbitals or p overlapping of two p orbitals.
orbitals or one s and one p orbital.

2. Its bond orbitals consists of a single Its bond orbital consists of two electron
electron cloud symmetrical about clouds, one above and other below the
internuclear axis. plane of participating atoms.

3. Now since the overlapping is along the The overlapping of orbitals is along the
axis, it is maximum and hence the sideways which is only partial and
bond formed is strong. The energy of a hence the bond formed is weak. The
carbon-carbon single () bond is 80 energy of a carbon-carbon pi bond is
kcal. only about 65 kcal.

4. The s electrons are referred as In  bond the electron ( electrons) are


localized. held less firmly and can be more easily
dissociated or polarised (i.e. attracted
to either end of the M.O.) by external
charge and hence the  electron are
referred as mobile electrons.

5. Sigma bonds are less reactive. Pi bonds are more reactive.

6. Sigma bond leads to expected distance Pi bonding leads to shortening of


between the two carbon atoms. distance between the two atoms
involved, e.g, the 1.34 Å and 1.20 Å
respectively.

7. Provided the groups or atoms are Due to resistance to rotation around the
smaller in size, they can undergo  bond the groups attached to it are not
torisonal rotation about single () free to rotate.
bonds.

8. The shape of the molecule is Pi bonding does not affect the shape of
determined by the s bonds present in the molecule in which it occurs.
the molecule.

9. -Bond can have independent -Bond always exists along with a


existence.  -bond.

(f) Characteristics of covalent compounds: (i) Covalent compounds have low melting points
and boiling points. As such they may be gases, liquids or low melting solids. (ii) They are
insoluble in water but soluble in organic (non-polar) solvents (iii) They do not conduct electricity
in the solution or the molten state. (iv) Since covalent bonds are rigid and possess directional
characteristics, therefore, they show stereoisomerism. (v) Their reactions are slow and
molecular in nature and never proceed to completion.

[ 18 ]
Periodic Classification of Elements and Chemical Bonding

Co-ordinate covalent, Co-ordinate or Dative bond: This is a special type of covalent bond.
Here, although both the electrons forming the bond are contributed only by one atom, these are
shared by both the atoms. The co-ordinate bond formation involves one sides sharing of a lone-
pair of electrons. The atom that donates the electron pair is known as donor and the atom which
accepts the electron pair is called the acceptor. The dative bond is shown by means of an arrow
() pointing away from the donor atom to the acceptor atom. Common examples are SO2, H2O2,
H2SO4, NH4+, SO3, etc.
+
H H H
·× ·×
H ×· N : + H+ H ×· N : H+ or H –· N  H
·×
·× H H
H

Ammonia (having lone


pair of electrons on N)

Some examples for compounds with more than one type of bond

NaOH, KOH, Na2CO3 (Ionic, covalent)


CO, NH3, BF3, [Co(NH3)3Cl3] Covalent, dative
NH4Cl, CuSO4, K4[Fc(CN)6], [Cu (NH3)]SO4 Ionic, covalent, dative

Illustration 1: Hydrogen cyanide, HCN


(i) Total number of valence electrons in HCN = 1+ 4 + 5 = 10
(ii) Writing the symbolic formula of HCN
H–CN
(iii) Completion of the duplet of H and octet of N.
··
H–C–N
··
(iv) Note that all the 10 electrons are accounted for, but still C is electron deficient. So
it is suggested that C and N may have multiple bond, viz. triple bond.
· ·
H –C  N
Illustration 2: Nitrogen trifluoride, NF3
(i) Total number of electrons = 5 + (3 × 7) = 26
(ii) Symbolic formula of NF3
F – N– F
|
F
(iii) Completion of octet of each fluorine atom.
·· ··
:F – N – F :
·· ··
:F:
··
(iv) Remaining 2 electrons are placed on N and thus, we have the following Lewis
formula for NF3.
·· ·· ··
:F – N – F :
·· ··
:F:
··
[ 19 ]
Periodic Classification of Elements and Chemical Bonding

Explanation of the failure of octet rule:

1. Sudgen’s view of singlet linkage: According to Sudgen, the octet rule is never violated but
in certain molecules the atoms are linked not only by covalent bonds but also by other type
of linkage known as singlet linkage (single electron linkage, half bond or simply singlet).
Singlet linkage is formed by one sided sharing of only one electron between the two atoms;
it is represented by a half arrow ( ) with its head pointing from the donor towards the
acceptor. Thus in SF6, there are only 2 covalent bonds and 4 singlet linkages.
··
:F: · ·
:F · · F: F
· · ××× · F F
S S
: F ·××· × · F : F F
· · :F:
·· F

SF6 molecule (note the presence of 8 electrons around S and each F atom).

2. Concept of overlapping of atomic orbitals: According to this principle, the presence of d


orbitals is responsible for the deviation from octet rule. For example, formation of phosphorus
pentachloride can be shown as below.
3s 3p 3d
P atom in the ground state
   

P atom in the excited state     

Thus in the excited state P atom has 5 unpaired electrons and hence forms 5 covalent bonds
in deviation from the octet rule.

Hybridisation
Merging (mixing) of dissimilar orbitals of almost similar energies to form new orbitals is known
as hybridisation and the new orbitals formed are known as hybrid orbitals. Following points are
important in the study of hybridisation.

(i) Hybridisation is not a real physical process, but is a concept which has been introduced to
explain some structural properties which could not be explained by simple valence bond theory.

(ii) It is the orbitals which undergo hybridisation and not electrons.

(iii) Only orbitals of similar energies and belonging to the same atom or ion undergo hybridisation.

(iv) The number of hybrid orbitals produced is equal to the number of pure orbitals mixed during
hybridisation, e.g., four hybrid orbitals are produced by mixing one s and three p (px, py and
pz) pure orbitals.

(v) Most of the hybrid orbitals are similar, i.e., they have identical shape and energy.

(vi) Hybrid orbitals lead to the formation of bonds known as hybrid bonds, which are stronger than
the non-hybrid bonds of comparable length.

[ 20 ]
Periodic Classification of Elements and Chemical Bonding

(vii) Hybrid orbitals, follow Hund’s rule and Pauli’s principle.


(viii) Hybridised orbitals are dumb-bell type; one loop larger and one smaller.
(ix) Hybridized orbitals show only head on overlapping and thus form only sigma bonds.
(x) The hybrid orbitals are distributed in space as far apart as possible. Different types of
hybridisation lead to different orientation in space. For example.
Type of hybridisation Orientation
sp hybridisation 2 hybrid orbitals at 180º
sp2 hybridisation 3 hybrid orbitals at 120º
sp3 hybridisation 4 hybrid orbitals at 109º 28'
The angle of hybrid orbitals is derived by cos  = – (1/x) where x is number of p orbitals
involved is hybridisation. Thus,
for sp cos  = –1   = 180º
1
for sp2 cos  = –   = 120º
2
1
for sp3 cos  = –   = 109º 28'
3
(xi) The order in which orbitals of a hybrid set are written in the order of increasing n, Main group
elements use the ns, np and nd orbitals and thus here hybridisations are written as sp3d, sp3d2,
sp3d3. On the other hand, transition elements use (n–1) d, ns and np orbitals and thus
hybridisation is written as dsp3, d2sp3.
1. sp3 Hybridisation (Tetrahedral hybridisation): This type of hybridisation involves mixing of
one s and three pure p orbitals to give four equivalent sp3 hybrid orbitals. These orbitals
are directed towards the four corners of a regular tetrahedron and separated by an angle of
109º28' (the normal tetrahedral angle).

H 

+ + +  H
X H
H 

2s 2px 2py 2pz


3
H
sp hybridisation

C

109º28
H H
H

[ 21 ]
Periodic Classification of Elements and Chemical Bonding

This type of hybridisation takes place in the formation of alkanes (methane, etc., saturated
organic compounds, in general), SiH4, CCl4, SiCl4, SnCl4, diamond, silica, SiC, NH4+, BF4–,
H3O+, NH3, H2O, SO42–, ClO4–, ClO3–, ClO2–, ClO–, etc.

Formation of methane. In the formation of methane each of the sp3 hybrid orbitals overlaps
axially with the 1s orbital of the hydrogen atoms to form four bonds between C and H .

In ammonia, since only three hydrogens overlap with the three sp3 hybrid orbitals, the fourth
sp3 hybrid orbital has a lone pair of electrons which repels the electrons involved in bond
formation with the result the tetrahedral bond angle of 109º 28' decreases to 107º 45'.
Similarly, since water has two lone pair of electrons, repulsion between lone pair and bond
pair electrons is more than in ammonia, and hence bond angle is further distorted to 104º 27'.

Chlorate ion (ClO3–) is also an example where the central atom is sp3 hybridised and has a
lone pair of electrons.

2. sp2 or Trigonal hybridisation: This type of hybridisation involves mixing of one s and two p (px
and py) orbitals to give three equivalent sp2 hybrid orbitals. The three sp2 hybrid orbitals are
directed towards the three corners of an equilateral triangle with an angle of 120º called as plane
trigonal.

The effective size of sp2 orbitals is slightly smaller than that of sp3 hybrid orbitals. Note that
an sp2 hybridised carbon atom has three sp2 hybrid orbitals and one pure p-orbital.

This type of hybridisation takes place in the formation of alkenes (i.e., C = C), benzene, boron
halides, AlCl3, SO2, SO3 graphite, NO3–, CO32–, etc.

+ +

2px 2py 2pz


2s
2
sp hybridisation

2
sp
120º

sp2 sp2

3. sp or Diagonal hybridisation: This type involves the intermixing of one s and one p (say
px) orbitals to give two equivalent hybrid orbitals, known as sp hybrid orbitals. The two sp
hybrid orbitals are directed diagonally, i.e., in a straight line with an angle of 180º (collinear
orbitals). The other two p orbitals (say py and pz) remain pure.

2s 2px
sp hybridisation

180º 180º
sp sp sp
sp sp

[ 22 ]
Periodic Classification of Elements and Chemical Bonding

sp-Hybrid orbitals are smaller in size than the sp3 as well as sp2 hybrid orbitals. Note that
an sp hybridised carbon has two sp hybrid orbitals and two pure p orbitals. This type of
hybridisation takes place in the formation of alkynes. (i.e, C  C), BeX2, BeH2, HCN,
CO2, N2, etc.

Hybridisation involves not only s and p orbitals, but d too. Thus we may sp3d, sp3d2 and sp3d3
hybridisation giving trigonal bipyramidal, octahedral and pentagonal bipyramidal geometry
respectively to the resulting molecule.

4. sp3d Hybridisation (Trigonal bipyramid hybridisation) : The mixing of one s, three p and one
d orbitals to form five hybrid orbitals of equal energy is known as sp3d hybridisation. The formation
of gaseous molecule of phosphorus pentachloride is explained by sp3d hybridisation. The electronic
configuration of the outer shell of the central atom phosphorus in the ground and excited states
is as follows:

P15 in ground state : [Ne] 3s2 3p3


P15 in excited state : [Ne] 3s1 3p3 3d1

Thefive sp3d hybrid orbitals (obtained by mixing of one s, three p and one d of the third shell)
overlap with the spz orbital of five chlorine atoms to give a trigonal structure.

It is important to note that all the five hybrid orbitals are not equivalent, three of these are
oriented to the corners of an equilateral triangle making an angle of 120º between them. The
bonds formed by these orbitals are called equitorial bonds. The other two orbitals are oriented
at right angles to the plane of the first set of three hybrid orbitals. The bonds formed by these
orbitals are called axial bonds. PCl5, thus, has a trigonal bipyramidal structure with two types
of bond lengths and bond angles.
Cla
Cle

90º
120º
Cle P
2.04Å

2.19Å Cle

Cla

(i) P – Claxial > P – Clequ (ii) Cle – P – Cle = 120º and Cla – P – Cle = 90º

Since, PCl5 is not completely a regular structure as its some bond angles are 90º and others
are 120º, it is highly reactive and splits in the solid state into PCl4+ and PCl6– ions having
tetrahedral and octahedral structures, respectively. Other examples are PF5 and SbCl5.

Xenon difluoride (XeF 2) also has sp3d hybridisation, the three equitorial positions being
occupied by three lone pair of electrons. Similarly, in ClF 3 (T-shape) and SF 4 (irregular
tetrahedral) two and one positions are occupied by lone pair of electrons.

5. sp3d2 Hybridisation (Octahedral hybridisation): In sulphur haxafluoride, SF6, the sulphur


atom is excited to give six unpaired electrons.

[ 23 ]
Periodic Classification of Elements and Chemical Bonding

S16 in ground state [Ne] 3s2 p4


S16 in excited state [Ne] 3s1 p3d2

The six outer electrons hybridise to produce six sp3d2 hybrid orbitals; the hybridization is known
as sp3d2 hybridization.

The six hybrid orbitals (sp3d2) of sulphur are equivalent and oriented in space towards the
corners of a regular octahedron. As a result all the six sulphur to fluorine bond distances
in SF6 are equal and all the angles are of 90º. Other examples are SeF6, TeF6, etc.

IF5 and XeF4 also have sp3d2 hybridisation, in the former one position and in the later two
positions are occupied by lone pair of electrons.

In some cases, the two d orbitals involved during hybridisation come from a lower principal
shell than the s – and p – orbitals; this type of hybridisation is known as d2sp3 hybridisation.

6. sp3d3 Hybridisation, e.g., IF7. The seven sp3d3 hybrid orbitals are not equivalent. There
are two sets of equivalent orbitals five of these are of one kind and directed towards the
vertices of a regular pentagon making an angle of 72º. The other two are directed at right
angles to the plane of the first set of hybrid orbitals. The molecule thus acquires pentagonal
bipyramidal geometry with two types of bond lengths and bond angles:

(i) I – Fa > I – Fe (ii) Fe – I – Fe = 72º and  Fa – I – Fe = 90º

Valence shell electron pair repulsion (VSEPR) theory: Like electrons, the orbitals
containing electrons also experience electrostatic repulsion from one another. Therefore, the
orbitals tend to keep apart from one another as far as possible so that they may experience
minimum electrostatic repulsion and hence have minimum energy and maximum stability of
the molecule. This forms the basis of valence shell electron pair repulsion theory
(VSEPR theory). According to this theory “the orbitals occupied by electron in the valence
shell of the central atom should be arranged in space in a way that they lie as far away
from one another as possible to impart maximum stability to the molecule”. It is important
to note that the arrangement of orbitals in space (orientation of orbitals in space) to give
maximum stability is applicable to molecular orbital electrons, i.e., bond pair of electrons
commonly abbreviated as bp as well as to lone pair of electrons (lp) present in the valence
shell of the central atom. However, the repulsion between different types of electron pair is
not same in magnitude but in the following order.

lp – lp Repulsion >lp – lp Repulsion > bp – bp Repulsion

Thus according to VSEPR theory, the shape (geometry) of a covalent molecule depends upon:

(a) the number of electron pairs (orbitals) present in the outer (valence) shell of the central atom.

(b) the number of bond pairs of electrons.

(c) the number of lone pairs of electrons.

Illustration 3: The ratio of  to -bonds in mesitylene is:


(a) 3 (b) 7
(c) 5 (d) 6

[ 24 ]
Periodic Classification of Elements and Chemical Bonding

Solution: (b) The structure of mesitylene is


H
H H

H H

H H
H H
Total number of -bonds = 21
Total number of -bonds = 3

Illustration 4: The number and type of bonds between two carbon atoms in calcium carbide are:
(a) one sigma, one pi (b) one sigma, two pi
(c) two sigma, one pi (d) two sigma, two pi
2-
Solution: (b) Calcium carbide is an ionic compound having anion C C . It has one sigma
and two pi bonds.

Illustration 5: In which of the following molecules / ions are all the bonds not equal?
(a) SF4 (b) SiF4
(c) XeF4 (d) BF4

Solution: (a) In SF4, sulphur undergoes sp3d hybridisation. It contains two axial and two equatorial
positions occupied by fluorine atoms and one equatorial position is occupied by lone
pair of electrons. Thus, bond lengths of equatorial positions are different than the bond
length of the axial positions.

Illustration 6: Which of the following have undistorted octahedral structures?


I. SF6 II. PF6
III. SiF62 IV. XeF6
Solution: (c) XeF6 has distorted octahedral structure. Xenon is XeF6 undergoes sp3d3 hybridisation
giving pentagonal bipyramidal with one axis is occupied by lone pair.

SAMPLE PROBLEMS 1.3 (MCQ)


Problem 1: The hybrid states of carbon in diamond, graphite and acetylene are respectively:
(a) sp2, sp, sp3 (b) sp, sp2, sp3
(c) sp3, sp2, sp (d) sp2, sp3, sp

Solution: (a) In diamond, each carbon is sp3-hybridised, in graphite it is sp2 and in acetylene is sp-
hybridised.

Problem 2: In sp3d3 hybridisation, which d-orbitals are involved?


(a) d x 2  y 2 , d z 2 , d xy (b) d x 2  y 2 , d xy , d yz
(c) d z 2 , d yz , d xy (d) d xy , d yz , d zx

Solution: (d)

[ 25 ]
Periodic Classification of Elements and Chemical Bonding

Problem 3: Molecule having trigonal bipyramidal geometry has sp3d hybridisation. Which d-orbitals is
involved?
(a) d xy (b) d yz
(c) d x 2  y 2 (d) d z 2

Solution: (d)

Problem 4: Out of N2O, SO2, I3 , I3 , H2O, NO 2 , N3 , the linear species are:
(a) NO 2 , I3 ,H 2 O (b) N 2 O, I3 , N 3
(c) N 2 O, I3 , N3 (d) N3 , I3 , SO2

Solution: (c) I

N IN O
N2O
N3 N N N

Problem 5: The bond dissociation energy of B — F in BF3 in 646 kJ mol–1 whereas that of C — F
in CF4 is 515 kJ mol–1. the correct reason for higher B—F bond dissociation energy is
compared to that of C — F is:
(a) smaller size of B atom as compared to that of C atom.
(b) stronger  bond between C and F in CF4
(c) significant p-p interaction between B and F in BF3 whereas there is no possibility
of such interaction between C and F CF4.
(d) lower degree of p-p interaction between B and F in BF3 than that between C and
F in CF4.

Solution: (c) In BF3, B is sp3 hybridised and has a vacant 2p orbital which overlaps laterally with
a filled 2p orbital of F forming strong p-p bond. However in CF4 the C does not
have any vacant p-orbital.

Problem 6: Number of bond pairs and lone pairs around the central atom in the I3 ion are:
(a) 2, 2 (b) 3, 2
(c) 2, 3 (d) 4, 3

Solution: (c) There are two bonding pairs and three lone-pairs in the outershell of central atom.
I

[ 26 ]
Periodic Classification of Elements and Chemical Bonding

Problem 7: Specify the co-ordination geometry around and hybridisation of N and B atoms in the
1 : 1 complex of BF3 and NH3:
(a) N : tetrahedral, sp3, B : tetrahedral, sp3 (b) N : pyramidal, sp3, B : pyramidal, sp3
(c) N : pyramidal, sp3, B : planar, sp2 (d) N : pyramidal, sp3, B : tetrahedral, sp3

Solution: (a) The lone pair present on nitrogen is ammonia molecule is donated to boron as BF3
is an electron deficient molecule forming a co-ordinate bond.

H F
H N B F
H F

Both N and B are in sp3-hybrid state, i.e.,. two tetrahedral are joined with each other.

Problem 8: Identify the correct order of decreasing bond strength in the bonds formed by
2s – 2s, 2p – 2p and 2s – 2p overlapping:
(a) 2s – 2s > 2s – 2p > 2p – 2p (b) 2s – 2s > 2p – 2p > 2s – 2p
(c) 2p – 2p > 2s – 2p > 2s – 2s (d) 2p – 2p > 2s – 2s > 2s – 2p

Solution: (c) Bond strength depends on the extent of overlapping. Maximum overlapping occurs in
the case of p-orbitals.
Problem 9: The maximum number of 90° angles between bond pair-bond pair of electrons is
observed in:
(a) sp3d2 hybridisation (b) sp3d-hybridisation
(c) dsp3-hybridisation (d) dsp2-hybridisation

Solution: (a) sp3d2 hybridization gives octahedral structure having all bonds at 90°.

Problem 10: The correct decreasing order of bond angles is:


(a) ClF3 > PF3 > NF3 > BF3 (b) BF3 > PF3 > NF3 > ClF3
(c) BF3 > NF3 > PF3 > ClF3 (d) BF3 > ClF3 > PF3 > NF3
Solution: (c) BF3 NH3 PF 3 ClF3
Structure Triangular Pyramidal Pyramidal T-shaped
planar
Bond angle 120° 106° 101° 90° and 180°

1.4 Molecular Orbital Theory for the Formation of Covalent Bond


According to this theory a covalent bond is formed when two half-filled orbitals of the two atoms
come nearer and then overlap each other to form a new bigger orbital, known as molecular
orbital (MO). The molecular orbital surrounds the atomic nuclei of both the atoms and each of
the electrons, which are now paired in the molecular orbital, is electrostatically attracted by both
the nuclei. It is important to note that energy of the MO is less than the sum of energies of the
two atomic orbitals and consequently the resulting molecule is more stable than the two separate
atoms. Like atomic orbitals, MOs also obey Pauli’s exclusion principle and thus one MO can
accommodate only two electrons with antiparallel spins.

[ 27 ]
Periodic Classification of Elements and Chemical Bonding

When two atomic orbitals overlap each other, two MOs are produced, namely bonding (having
lesser energy than the energies of the separate atomic orbitals) and antibonding (having higher
energy than the energies of the two separate atomic orbitals). These two new MO’s spread over
both the atoms and either may contain two electrons.

If overlap of the two atomic orbitals takes place along their axes, the resulting bonding MOs are
known as sigma () orbitals and the bond formed as a  bond. On the other hand, if overlapping
of the two atomic orbitals takes place sideways, the resulting MOs are known as pi ( orbitals
and the bond formed by them as  bond. -Bond is formed by the overlapping of py – py orbitals.

According to the principle of Linear combination of Atomic Orbitals (LCAO), bonding MO


is formed by addition of wave functions of the electron waves of atomic orbitals while antibonding
MO is formed by substraction of the wave functions of the atomic orbitals.

(i) The bonding MO usually contains a pair of electrons in the ground state of the molecule and
hence contributes towards stability of the molecule. In contrast, electrons present in the
antibonding MO’s contribute to the repulsion between two atoms. In other words, the energy
of the bonding MO is less than those of the combining AO’s while the energy of the
antibonding MO is more than those of combining AO’s.

(ii) Only AO’s of comparable energies and proper orientations combine to form MO’s. For
example, 1s can combine with 1s and not with 2s. Similarly, if Z-is the molecular axis, scan
combine with pz but not with px or py, or pz can combine with pz but not with px or py to form
-molecular orbitals.

(iii) The bonding MO’s are represented as , ,.....etc. while the corresponding antibonding MO
are represented as , , etc. Thus 1s-atomic orbitals of two atoms combine to form two
molecular orbitals labelled as  (2s) (bonding) and * (2 s) (antibonding). Of the 2 p-orbitals,
if Z-is the molecular axis, two pz -orbitals from - orbitals, i.e.,  (2 pz) while 2px and 2py-
orbitals from -bonds, i.e.,  (2 px), (2 px), (2py) and  (2py).

(iv) The shapes of MO’s formed depend upon the type of the combining AO’s.

(v) The filling of the MO’s occurs according to the same principle as applicable to filling of AO’s.
i.e., Aufbau principle (MO’s are filled in order of their increasing energies), Pauli exclusion
principle (a MO can have a maximum of two electrons and these must have opposite spins)
and Hunds rule of maximum multiplicity (pairing of electrons in degenerate MO’s does not
take place until each one of the them has one electron with the same spin).

All the electrons occupying bonding MO’s (N b) and antibonding MO’s (N a) are counted
separately & if

(a) Nb > Na then the molecule will be stable.

(b) Nb < Na or Nb = Na the molecule is unstable.

Bond order (number of covalent bonds present in a diatomic molecule). Mathematically,

[ 28 ]
Periodic Classification of Elements and Chemical Bonding

1
Bond order = [Nb – Na]
2
where, Nb = No. of electrons in bonding MO’s.

Nb = No. of electrons in antibonding MO’s.

Important points regarding bond order

(i) Bond order of 1, 2 or 3 corresponds to a single, double or triple bond respectively.

(ii) Bond order may be fractional and even zero. A zero bond order indicates the equal number
of bonding and antibonding electrons. Zero or less than zero value indicates that the species
1
does not exist, while the fractional value, e.g. + value indicates that the species exists but
2
it is unstable, e.g., H2+ and He2+.

*(2p)x
*(2p)x

2p *
(2p) * ()
y 
2p 2p * * 2p
z (2p)
y 
z

(2p)y (2p)z (2p)y 


z

(2p)x
Increasing enerhgy

Increasing enerhgy

(2p)x (2p)y (2p)z

*
(2s) *
(2s)
2s 2s 2s 2s

(2s)
(2s)

*
(1s) *
(1s)
1s 1s 1s 1s
(1s)

Molecular Atomic (1s)


Atomic
orbitals orbitals orbitals AOs MOs AOs
(iii) The bond order of a molecule is directly proportional to its bond dissociation energy and
inversely proportional to its bond length. For example,

Molecule Bond order Bond length Bond diss. energy (kJ mol–1)

F2 (F – F) 1 142 pm 158
O2 (O = O) 2 121 pm 498
N2 (N  N) 3 110 pm 945

Magnetic character. If all the electrons in a molecule are paired, the substance is diamagnetic;
while when the molecule has one or more electrons unpaired, it will be paramagnetic.

Illustration 1 Hydrogen molecule (H2)

Total number of electrons in 2 H atoms = 1 + 1 = 2

[ 29 ]
Periodic Classification of Elements and Chemical Bonding

Electronic configuration of H2 molecule: =  1s2

1 1
 Bond order = (Nb – Na) = (2 – 0) = 1
2 2
Illustration 2: Hydrogen molecule ion, H2+
Number of electrons in H2+ = 2 – 1 = 1
Electronic configuration =  1s1
1 1
 Bond order = (1 – 0) =
2 2
1
Since the bond order is , H2+ ion can exist but it is unstable. Further as it has 1 unpaired
2
electron, it is paramagnetic.
Illustration 3: Nitrogen (N2) : The electronic configuration of nitrogen atom shows that it has 5 valency
electrons, hence the molecular orbitals of nitrogen molecule has 10 electrons. These 10
electrons are distributed in the following manner:
KK, ( 2s)2 (*2s)2 (y 2p)2 (z 2p)2 ( 2px)2
Thus the molecular orbital of nitrogen has 8 electrons in bonding and 2 electrons in
antibonding orbitals. Thus the bond order or the number of bonds in nitrogen molecule may
be calculated as below:
Bond order = 1/2 [No. of bonding electrons – No. of antibonding electrons]
= 1/2 [8 – 2] = 1/2 × 6 = 3
Illustration 4: Oxygen (O2) : Oxygen (Z = 8) has the electronic configuration 1s2, 2s2p4, so 12 electrons
are to be filled in the molecular orbitals of oxygen (leaving aside the electrons of the K
shell). These 12 electrons are filled in the following manner.
KK, ( 2s)2 (*2s)2 ( 2px)2 ( 2py)2 ( 2py)2 (*2py)1 (*2pz)1
Note that the 11th and 12th electrons in the molecular orbital of oxygen go to *2py and
*2pz orbitals (Hund’s rule); it is because of the fact that these two antibonding orbitals
are of equal energy. Since, now there are two unpaired electrons, the oxygen molecule
is paramagnetic.
Bond order = 1/2 (No. of bonding electrons – No. of antibonding electrons)
= 1/2 (8 – 4) = 2
Hence, the two oxygen atoms in oxygen molecule are linked by a double bond.
Further, the presence of 4 antibonding electrons in oxygen molecule explain its less stability
(higher reactivity) than nitrogen molecule having only 2 antibonding electrons (bond energy
of O – O in O2 = 118 kcal mol–1; bond energy of N – N bond in N2 = 225 kcal mol–1).
Illustration 5: Molecular orbital configuration of O2+. Number of electrons in the bonding orbitals of
O2+ = 12 – 1 = 11
MO configuration : KK, ( 2s)2, (*2s)2 ( 2px)2 ( 2py)2 ( 2pz)2 (*2py)1 (*2pz)0

83 1
Bond order =  2
2 2
1
Bond order of  2 indicates that O2+ is more stable than O2. As O2+ has 1 unpaired
2
electron, it is paramagnetic.

[ 30 ]
Periodic Classification of Elements and Chemical Bonding

SAMPLE PROBLEMS 1.4 (MCQ)


Problem 1: Which of the following have identical bond order?
(I) CN– (II) O2
(III) NO+ (IV) CN+
(a) I, III (b) I, II
(c) II, IV (d) I, II, III

Solution: (a) Isoelectronic species have same bond order.


CN– = 6 + 7 + 1 = 14 electrons, O2 = 8 + 8 + 1 = 17 electrons,
NO+ = 7 + 8 – 1 = 14 electrons.
CN+ = 6 + 7 – 1 = 12 electrons.

Problem 2: Which of the following species is diamagnetic in nature?


(a) He2 (b) H2

(c) H 2 (d) H 2

Solution: (b) He2 : (σ1s) 2 (σ *1s)1 are electron - paramagnetic


H2 : H 2 : (σ1s) 2 no unpaired electron – diamagnetic
H 2 : (σ1s)1 one unpaired electron - paramagnetic
H 2 : (σ1s) 2 (σ *1s)1 one unpaired electron – paramagnetic

Problem 3: H2, Li2, B2 each has bond order equal to 1, the order of their stability is:
(a) H2 = K2 = B2 (b) H2 > Li2 > B2
(c) H2 > B2 > Li2 (d) B2 > Li2 < H2

Solution: (c) H2, Li2, B2 are not equally stable Li atom is much larger in size than H-atom. The
bond order in Li2 is much large than bond length in H2. Moreover, L2 molecule has
two electrons in the antibonding molecular orbital while H2 has no electrons in the
antibonding molecular orbital. Thus, Li2 is less stable than H2 (Bond energy of Li2 =
110 kJ mol–1 + Bonding energy of H2 = 438 kJ mol–1). Boron atom is smaller than
lithium atom but larger than hydrogen atom. The B2 is more stable than Li2 but less
than H2 (Bond energy of B2 = 290 kJ mol–1).

Problem 4: Which of the following pair is expected to have the same bond order?
(a) O2, N2 (b) O 2 , N 2

(c) O 2 , N 2 (d) O 2 , N 2

1
Solution: (b) O 2 , N 2 – Each has 15 electron. Bond order = (10  5) = 2.5
3
Problem 5: In a series, ethane, ethylene and acetylene, the C – H bond energy is:
(a) the same in all the three compound (b) greatest in acetylene
(c) greatest in ethylene (d) greatest in ethane

[ 31 ]
Periodic Classification of Elements and Chemical Bonding

Solution: (b)
H H H H
H C C H ; C C ; H C C H
H H H H
sp sp2 sp3
50%
31.3% 25%
Problem 6: The correct order of the O – O bond length in O2, H2O2 and O3 is:
(a) O3 > H2O2 > O2 (b) O2 > H2O2 > O3
(c) O2 > O3 > H2O2 (d) H2O2 > O3 > H2

Solution: (d) Bond between —O—O— in H2O2 in single bond (1.45Å). Bond between O—O in
O3 is intermediate between single and double bonds due to resonance (1.278Å) while
bond between O2 molecule is a double bond (1.21Å).

Problem 7: Molecular orbital electronic configuration for ‘x’ anion is:


KK(σ2s) 2 (σ * 2s) 2 (π2 p x ) 2 (π2 py ) 2 (σ2 pz ) 2 (π * 2 p x ) 2

(a) N 2 (b) O2


(c) N22 (d) O22

Solution: (a) N 2 (15 electrons). The given molecular orbital configuration cannot be assigned in

O2 (15 electrons) as in O2 molecule (2px) MO he lower energy then 2pz and
2py orbitals.

Problem 8: Arrange the following in order of their bond order:


O 2 , O 2 , O 2 and O22 :
(a) O 22 , O 2 , O 2 , O 2 (b) O 2 , O 2 , O 2 , O 22
(c) O 2 , O 2 , O2 , O 22 (d) O 22 , O 2 , O 2 , O 2

10  6 10  5
Solution: (d) B.O. of O2 =  2.0; B.O. of O 2   2.5
2 2

 10  7 10  8
B.O. of O 2   1.5; B.O. of O 22  1
2 2

Problem 9: N2 and O2 are converted to monopositive cations N 2 and O2 respectively. What is
incorrect?

(a) In N 2 , the N—N bond is weakened

(b) In O2 , the bond order increases

(c) In O2 , the paramagnetism decreases

(d) Both O2 and O2 are paramagnetic

[ 32 ]
Periodic Classification of Elements and Chemical Bonding

2 2 2 2 2 2
Solution: (d) N 2 : (σ2s) (σ * 2s) (π2 p x ) (π2 p y ) (σ2 pz ) (π * 2 p x )
10  4
B.O.  3
2
94
B.O.N 2   2.5
2
O 2 (σ2s) 2 (σ *1s) 2 (σ2 s) 2 (σ2s)2 (σ2 pz ) 2 (π2 p x ) 2 (π2 p y ) 2 (π * 2 p x ) 2
10  6
B.O.  2
2
10  5
B.O. O 2   2.5
2
Bond order < Bond energy = Strength of the bond

Polar and Non-polar Covalent Bonds

If the covalent bond is formed between similar atoms (H and H, Cl and Cl etc.), the shared pair
of electrons lies exactly in the middle and the electron cloud is uniformly distributed about the two
atoms. Such a covalent bond is called non-polar covalent bond. On the other hand, if a covalent
bond is formed between two different atoms, then the shared pair of electrons is displaced
towards the more electronegative atom. As a result, centres of positive and negative charges of
the system do not coincide. In other words, one end of the bond acquires a partial negative charge
(–) and the other end gets a partial positive charge (+). Such a bond is called polar covalent
bond. For example,
·· ··  –  –
H : H :Cl : Cl: H – Cl l – Cl
·· ··
Non-polar molecules Polar molecules

The extent of polar character of a bond is determined by the electronegativity (En) difference of
the bonding atoms.

Dipole Moment

For a diatomic molecule, the product of the magnitude of the positive or the negative charge on each
atom of the molecule and the distance between the centres of the two atoms is called dipole
moment, i.e.,
=q×d

where µ is the dipole moment, q is the charge and d is the distance. Its unit is Debye,
i.e., 10–18 esu cm.
H F H Cl H Br H I
= 1.98 D = 1.03 D = 0.78 D = 0.38 D

Percentage ionic character of a covalent bond. It depends upon the following two factors:

(a) Electronegativity difference.: Higher the electronegativity difference between the two atoms
forming the covalent bond, higher is the percentage of ionic character. For example,

Electronegativity difference 0.2 0.6 0.8 1.2 1.9 2.2 2.8 3.2
Percentage ionic character 1 10 15 30 50 70 80 92
Covalent character increases Ionic character increases
Nature of bond

[ 33 ]
Periodic Classification of Elements and Chemical Bonding

Several empirical equations have been proposed to calculate the percentage ionic character from
the electronegativity difference of the bonding atoms. Two such equations are:

Pauling Equation : % Ionic character = (1 – e–1/4 (xA – xB)) × 100

Hannay and Smith Equation: % Ionic character = 16 (XA – XB) + 3.5 (XA – XB)2

Here, XA and XB are the electronegativities of the two atoms forming a bond.

It may be mentioned here that both these equations, however, give approximate values.

(b) Dipole moment : Higher the dipole moment of a covalent bond higher is its percentage ionic
character. It can be calculated by the following equations:

Obtained dipole moment of the bond


% Ionic character = Dipole moment of a pure ionic bond  100

The dipole moment of a pure ionic bond is obtained by multiplying the electronic charge by the
distance between the two atoms, i.e,.,  = q × d where q = 4.8 × 10–10 esu and distance d is in
Å, i.e., 10–8 cm.

Illustration: Find out the percentage ionic character of a diatomic molecule having a dipole
moment of 1.98 D and bond length of 0.92 Å.

Solution: Dipole moment of the molecule if the bond were 100% ionic =  × d

= 4.8 × 10–8 esu × 0.92 × 10–8 cm = 4.42 × 10–18 esu – cm = 4.42 D

Actual dipole moment = 1.98 D

1.98 D  100
 Ionic character =  44.8%
4.42 D

[ 34 ]
Periodic Classification of Elements and Chemical Bonding

CHAPTER ASSIGNMENT
MULTIPLE CHOICE QUESTIONS (Single Correct)
1. The ions O2–, F–, Na+, Mg2+ and Al3+ are isoelectronic. Their ionic radii show:
(a) an increase from O2– to F– and then decrease from Na+ to Al3+
(b) a decrease from O2– TO F– and then increase from Na+
(c) a significant increase from O2– to Al3+
(d) a significant decrease from O2– to Al3+

2. Which of the following has maximum ionisation enthalpy?


(a) K (b) Na (c) Mg (d) Be

3. The electronic configuration of transition elements is exhibited by


(a) ns1 (b) ns2np 3 (c) ns2(n – 1)d10 (d) (n – 1)d1 – 10ns0 – 2

4. Amongst the following elements (whose electronic configuration are given below) the one having
highest ionisation energy is:
(a) [Ne]3s23p1 (b) [Ne]3s23p3 (c) [Ne]3s23p2 (d) [Ar]3d104s24p3

5. For electron affinity of halogens, which of the following is correct?


(a) Br > F (b) F > Cl (c) F > I (d) Br > Cl

6. Ionisation energy of hydrogen is:


(a) much higher than that of chlorine (b) slightly higher than that of chlorine
(c) smaller than chlorine (d) equal to that of chlorine

7. Which of the following is correct?


(a) The number of electrons present in the valence shell in SF6 is 12.
(b) The rates of reaction of ionic compounds are very slow.
(c) According to VSEPR, SnCl2 is a linear molecule.
(d) The correct order of ability to form ionic compounds among Na +, Mg2+ and Al3+ is
Al3+ > Mg2+ > Na+.

8. Which of the following is not a correct statement?


(a) Every AB3 molecule does in fact have square pyramidal structure
(b) Multiple bonds are always shorter than corresponding single bonds
(c) The electron deficient molecules act as Lewis acids
(d) The canonical structures have no real existence

9. The bond order of carbon in CO32 ion is:

(a) 1 (b) 2 (c) 1.33 (d) 1.5

10. How many resonating forms can be written for nitrate and chlorate ions respectively?
(a) 3, 2 (b) 3, 3 (c) 2, 3 (d) 3, 4

[ 35 ]
Periodic Classification of Elements and Chemical Bonding

11. Arrange the following molecules in order of decreasing bond angles:


SnCl4, BF3, BeCl2, H2O, SF6
(a) BeCl2 > BF3 > SiCl4 > H2O > SF4 (b) SF6 > H2O > SiCl4 > BF3 > BeCl2
(c) SiCl4 > BF3 > BeCl2 > H2O > SF6 (d) H2O > SiCl4 > SF6 > SF3 > BeCl2

12. Apply VSEPR theory to decide the shape of SF4 molecule. The shape of SF4 molecule is:
(a) tetrahedral (b) square planar (c) see-saw (d) none of these

13. Which of the following molecular species has unpaired electron(s)?

(a) N2 (b) F2 (c) O2 (d) O22

14. Using MO theory predict which of the following species has the shortest bond length?

(a) O22 (b) O2 (c) O2 (d) O22

15. Which resonance structure of N2O is more favourable than others?

+ +
(a) N N O (b) + (c) 2 N + (d) None of these
N N O N O

INTEGER TYPE QUESTIONS


16. Calculate the number of pairs of unshared electrons present in CO32 ion.

17. How many number of molecules among the following which do not satisfy octet rule?
BeCl2, AlCl3, H2O2, H2SO4, HNO3, SO3, PCl5, CO2, CO, O3, HClO4, NO 2
18. How many number of species among the following which have linear shape?
H2S, XeF2, NO2, N2O, I3 , ICl2 , Br3 , HgCl2, SnCl2, SO2, N3 , C2H4
19. Calculate the number of species among the following which have zero dipole moment.
SO2, CO2, NO2, BF3, CH4, NF3, p-C8H4Cl2, CH2Cl2, ICl2 , XeF4
20. Count the number of 90° bond angles present in the molecule of PCl5.

21. Calculate the number of species among the following which have fractional bond order.
Li2, He2 , N 2 , N 22 , O2 , O2 , CO32 , C 6 H 6 , O3, O22

22. Calculate the number of H2O molecules to which each H2O molecule is linked through hydrogen
bonds.

23. Calculate the number of pi bonds present in the molecule of anthracene.

24. Calculate the number of water molecule(s) directly bonded to the metal centre in CuSO4.5H2O is

25. Based on VSEPR theory, how many number of 90° angles are there in BrF5.

MULTIPLE CHOICE QUESTIONS (More One Correct)


26. In which of the following , the hybrid orbitals of the central metal atom have the same s-character?
(a) CH4 (b) XeO3 (c) Ni(CO)4 (d) [Ni(CN)4]2–

[ 36 ]
Periodic Classification of Elements and Chemical Bonding

27. Which one of the following molecule(s) is (are) expected to exhibit diamagnetic behaviour?
(a) O2 (b) S2 (c) C2 (d) N2

28. For resonance structure, a molecule should have


(a) identical arrangement of atoms (b) nearly same energy content
(c) the same number of paired electrons (d) identical bonding

29. CO2 is isostructural with


(a) HgCl2 (b) SnCl2 (c) C2H2 (d) NO2

30. Which of the following have identical bond order?


(a) CN– (b) O2 (c) NO + (d) CN+

31. The molecules that will have dipole moment is/are


(a) 2,2-dimethyl propane (b) trans-2-pentene
(c) cis-3-hexene (d) 2,2,3,3-tetramethyl butane

32. The factors which increase covalent character in an ionic compound are
(a) Small size of cation (b) Small size of anion
(c) Large charge on the cation (d) Small charge on the anion

33. The molecules or ions in which the central atom has only bond pairs of electrons are

(a) NH 4 (b) NO3 (c) SnCl2 (d) SF4

34. The molecules or ions which have bond pairs as well as lone pairs of electrons on the central
atom are:
(a) SF4 (b) ClF3 (c) XeF2 (d) CO32 ion

35. Resonating structures have


(a) widely different energies (b) same positions of atoms
(c) a real existence (d) same number of unshared electron

MATRIX MATCH TYPE QUESTIONS


36. Column - I Column - II

(A) I3 (p) sp2

(B) XeF2 (q) sp3d


(C) SnCl2 (r) Linear
(D) SO2 (s) V-shape

37. Column - I Column - II


2
(A) O 2
ion (p) Bond order 1.0

(B) O2 ion (q) Bond order 2.5

(C) N 2 ion (r) Paramagnetic

(D) N 2 ion (s) Diamagnetic

[ 37 ]
Periodic Classification of Elements and Chemical Bonding

COMPREHENSION TYPE QUESTIONS


Comprehension - I

To compare the polarities of different bonds and molecules, the term dipole moment was introduced.
Experimental methods are available to measure the dipole moments of different substances. Knowing the
experimental value of dipole moment, the percentage ionic character can be calculated.

38. The dipole moment of HBr is 0.78 × 10–18 cm and interatomic spacing is 1.41Å. The % ionic
character of HBr is
(a) 7.5 (b) 11.7 (c) 15 (d) 27

39. Which of the following has the highest dipole moment?


(a) AsH 3 (b) SbH3 (c) PH 3 (d) NH 3

40. Which of the following compounds has dipole moment approximately equal to that of chlorobenzene
(a) o-Dichlorobenzene (b) m-Dichlorobenzene
(c) p-Dichlorobenzene (d) p-Chloronitrobenzene
Comprehension - II

To explain the equivalence of 4 C-H bonds in CH4 or 3. B-F bonds in BF3 or 2 Be-F bonds in BeF2 etc., and
to explain the bond angles of such molecules and hence their shapes, the concept of hybridisation was put
forward.
41. The geometry of ClO3 ion according to Valence Sheel Electron Pair Repulsion (VSEPR) theory
will be
(a) Planar triangular (b) Pyramid
(c) Tetrahedral (d) Square planar

42. The hybridisation of atomic orbitals of nitrogen in NO 2 , NO3 and NH 4 are


(a) sp2, sp3 and sp2 respectively (b) sp, sp2 and sp3 respectively
(c) sp2, sp and sp3 respectively (d) sp2, sp3 and sp respectively

43. The AsF2 molecule is trigonal bipyramidal. The hybrid orbitals used by the As atoms for bonding are:
(a) d x 2  y 2 , d x 2 , s , p x , p y (b) d xy , s, px , p y , pz (c) s, px , p y , pz , d x 2 (d) d x 2  y 2 , s , p x , p y , p z

[ 38 ]
Periodic Classification of Elements and Chemical Bonding

PREVIOUS YEAR QUESTIONS


1. The element with the highest first ionization potential is? [IIT]
(a) Boron (b) Carbon (c) Nitrogen (d) Oxygen

2. The electronegativity of the following elements increases in the order [IIT]


(a) C, N, Si, P (b) N, Si, C, P (c) Si, P, C, N (d) P, Si, N, C

3. The first ionization potential in electron volts of nitrogen and oxygen atoms are respectively given
by [IIT]
(a) 14.6, 13.6 (b) 13.6, 14.6 (c) 13.6, 13.6 (d) 14.6, 14.6

4. Atomic radii of fluorine and neon in angstrom units are respectively given by [IIT]
(a) 0.72, 1.60 (b) 1.60, 1.60 (c) 0.72, 0.72 (d) none of these

5. The statement that is not true for the long form of the periodic table is [IIT]
(a) It reflects the sequence of filling the electrons in the order of sub-energy levels s, p, d and f
(b) It helps to predict the stable valency states of the elements
(c) It reflects trends in physical and chemical properties of the elements
(d) It helps to predict the relative ionicity of the bond between any two elements

6. The first ionization potential of Na, Mg, Al and Si are in the order [IIT]
(a) Na < Mg > Al < Si (b) Na > Mg > Al > Si (c) Na < Mg < Al > Si (d) Na > Mg > Al < Si

7. The outermost electronic configuration of the most electronegative element is [IIT]


(a) ns 2 np 3 (b) ns 2 np 4 (c) ns 2 np 5 (d) ns 2 np 6

8. Which of the following is smallest in size? [IIT]


(a) N 3– (b) O 2– (c) F – (d) Na +

9. Which one of the following is correct order of increase in size? [IIT]


(a) Mg < Na+ < F– < Al (b) Na+ < Al < Mg < F–
(c) Na+ < F– < Al < Mg (d) Na+ < F– < Mg < Al

10. The correct order of electron affinity of the elements of oxygen family in the periodic
table is [IIT]
(a) O > S > Se (b) S > O > Se (c) S > Se > O (d) Se > O > S

11. The statement that is not correct for the periodic classification of elements is [IIT]
(a) The properties of the elements are the periodic function of their atomic numbers
(b) Non-metallic elements are lesser in number than metallic elements
(c) The first ionization energies of elements along a period do not vary in a regular manner with
increase in atomic number
(d) For transition elements the d-subshells are filled with electrons monotonically with increase in
atomic numbers

[ 39 ]
Periodic Classification of Elements and Chemical Bonding

12. The decreasing order of the second ionization potential of K, Ca and Ba is [IIT]
(At. Nos. K = 19, Ca = 20, Ba = 56)
(a) K > Ca > Ba (b) Ca > Ba > K (c) Ba > K > Ca (d) K > Ba > Ca

13. Ionic radii of [IIT]


(a) Ti4+ < Mn2+ (b) 35
Cl– < 37Cl– (c) K+ > Cl– (d) P3+ > P5+

14. The correct order of radii is [IIT]


(a) N < Be < B (b) F– < O2– < N3– (c) N < Li < K (d) Fe3+ < Fe2+ < Fe4+

15. According to the Periodic Law of elements, the variation in properties of elements is
related to their
(a) nuclear masses (b) atomic numbers
(c) nuclear neutron-proton number ratio (d) atomic masses [IIT]

16. Which of the following are isoelectronic and isostructural? [IIT]


NO3–, CO32–, ClO3–, SO3
(a) NO3 –, CO32– (b) SO3, NO3– (c) ClO3–, CO32– (d) CO22–, SO3

17. If molecule MX3 has zero dipole moment, the sigma bonding orbitals used by M (Atomic
number < 21) are [IIT]
(a) pure p (b) sp hybrid (c) sp2 hybrid (d) sp3 hybrid

18. The pair of molecules forming strongest hydrogen bonds are [IIT]

(a) SiH6 and SiF6 (b) CH3– C –CH3 and CHCl3


||
O
(c) H– C –OH and CH3 – C –OH (d) H2O and H2
|| ||
O O
19. The types of bonds present in CuSO4·5H2O are [IIT]
(a) electrovalent and covalent only
(b) electrovalent and co-ordinate covalent only
(c) electrovalent, covalent, co-ordinate covalent and hydrogen bonds
(d) covalent and co-ordinate covalent only

20. Which of the following compound has a linear structure? [IIT]


(a) CCl4 (b) SO2 (c) C2H 2 (d) C2H 4

21. The species which has pyramidal shape is [IIT]


(a) PCl3 (b) SO3 (c) CO3 2– (d) NO3 –

22. Which of the following species is paramagnetic? [IIT]


(a) O2 – (b) CN– (c) CO (d) NO+

23. The molecule which has zero dipole moment is [IIT]


(a) CH2Cl2 (b) BF 3 (c) NF 3 (d) ClO2

[ 40 ]
Periodic Classification of Elements and Chemical Bonding

24. Which of the following has zero dipole moment? [IIT]


(a) ClF (b) PCl3 (c) SiF4 (d) CFCl3

25. Among the following species, identify the isostructural pairs [IIT]
NF3, NO3–, BF3, H3+O, HN3
(a) [NF3, NO3–] and [BF3, H3+O] (b) [NF3, HN3] and [NO3–, BF3]
(c) [NF3, H3+O] and [NO3–, BF3] (d) [NF3, H3+O] and [HN3, BF3]

26. KF combines with HF to form KHF2. The compound contains the species [IIT]
(a) K+, F– and H+ (b) K+, F– and HF (c) K+ and [HF2]– (d) [KHF]+ and F2

27. Among the following compounds the one that is polar and has the central atom with sp 2
hybridisation is [IIT]
(a) H2 CO 3 (b) SiF4 (c) BF 3 (d) HClO2

28. Which contains both polar and non polar bonds? [IIT]
(a) NH4Cl (b) HCN (c) H2 O 2 (d) CH4

29. Which of the following involve sp2 hybridisation? [IIT]


(a) CO2 (b) SO2 (c) N 2 O (d) CO

30. The hybridization of atomic orbitals of nitrogen in NO2+, NO3– and NH4+ are [IIT]
(a) sp2, sp3 and sp2 respectively (b) sp, sp2 and sp3 respectively
(c) sp2, sp and sp3 respectively (d) sp2, sp3 and sp respectively

31. Among H2O, H2S, H2Se and H2Te, the one with the highest boiling point is [IIT]
(a) H2O because of hydrogen bonding (b) H2Te because of higher molecular weight
(c) H2S because of hydrogen bonding (d) H2Se because of lower molecular weight

32. Which of the following molecular species has unpaired electron(s)? [IIT]
(a) N 2 (b) F 2 (c) O 2 – (d) O 2 2–

33. Which of the following hydrocarbons has the lowest dipole moment? [IIT]

H 3C CH3
(a) C (b) CH3 CCCH3 (c) CH3 CH2 CCH (d) CH2 =CH–CCH
H H

34. The species having bond order different from that in CO is [IIT]
– + –
(a) NO (b) NO (c) CN (d) N 2

35. The percentage of p-character in the orbitals forming P. P bonds in P4 is [IIT]


(a) 25 (b) 33 (c) 50 (d) 75

36. Hyperconjugation involves overlap of the following orbitals [IIT]


(a)  (b) p (c) p – p (d) 

[ 41 ]
Periodic Classification of Elements and Chemical Bonding

37. STATEMENT-1: The plot of atomic number (y-axis) versus number of neutrons (x-axis) for stable
nuclei shows a curvature towards x-axis from the line of 45° slope as the atomic number is increased.
and
STATEMENT-2: Proton-proton electrostatic repulsions begin to overcome attractive forces involving
protons and neutrons in heavier nuclides. [IIT]
(a) Statement-1 and 2 is True; Statement-2 is a correct explanation for Statement-1
(b) Statement-1 and 2 is True; Statement-2 is NOT a correct explanation for Statement-1
(c) Statement-1 is True, Statement-2 is False
(d) Statement-1 is False, Statement-2 is True
38. The species having pyramidal shape is [IIT]

(a) SO3 (b) BrF 3 (c) SiO 32 (d) OSF2

39. Assuming that Hund’s rule is violated, the bond order and magnetic nature of the diatomic
molecule B2 is [IIT]
(a) 1 and diamagnetic (b) 0 and diamagnetic
(c) 1 and paramagnetic (d) 0 and paramagnetic

40. Match each of the diatomic molecules in Column I with its property/properties in Column II.

Column I Column II [IIT]

A. B 2 (p) Paramagnetic

B. N 2 (q) Undergoes oxidation


C. O 2 – (r) Undergoes reduction

D. O 2 (s) Bond order  2


(t) Mixing of ‘s’ and ‘p’ orbitals

41. The oxidation number of Mn in the product of alkaline oxidative fusion of MnO2 is [IIT]

42. Based on VSEPR theory, the number of 90 degree F – Br – F angles in BrF5 is [IIT]

43. Among the following, the number of elements showing only one non-zero oxidation state is
O, Cl, F, N, P, Sn, Tl, Na, Ti. [IIT]

44. The periodic table consists of 18 groups. An isotope of copper, on bombardment with protons, undergoes
a nuclear reaction yielding element X as shown below. To which group , element X belongs in the
periodic table?
63
29 Cu 11 H 
610 n   211 H  X [IIT]

45. The shape of XeO2F2 molecule is [IIT]


(a) trigonal bipyramidal (b) square planar (c) tetrahedral (d) see-saw

46. EDTA4– is ethylenediaminetetraacetate ion. The total number of N – Co – O bond angles in


[Co(EDTA)]–1 complex ion is [JEE-Advance]

[ 42 ]
Periodic Classification of Elements and Chemical Bonding

47. Hydrogen bonding plays a central in the following phenomena: [JEE-Advance]


(a) Ice floats in water
(b) Higher Lewis basicity of primary amines than tertiary amines in aqueous solutions
(c) Formic acid is more acidic than acetic acid
(d) Dimerisation of acetic acid in benzene

48. A list of species having the formula XZ4 is given below:


XeF4, SF4, SiF4, BF4 , BrF4 , [Cu(NH3)4]2–, [CoCl4]2– and [PtCl4]2–
Defining shape on the basis of the location of X and Z atoms, the total number of species having a
square planar shape is. [JEE-Advance]

49. Match the orbital overlap figures shown in List - I with the description given in List-II and select
the correct answer using the code given below the list. [JEE-Advance]
List I List II

P. 1. p – d  antibonding

Q. 2. d – d bonding

R. 3. p – d bonding

S. 4. d – d antibonding

(a) A-2; B-1; C-3; D-4 (b) A-4; B-3; C-1; D-2
(c) A-2; B-3; C-1; D-4 (d) A-4; B-1; C-3; D-2

50. AlCl3 is covalent while AlF3 is ionic. This fact can be justified on the basis of [DCE]
(a) valence bond theory (b) crystal structure (c) lattice energy (d) Fazan’s rule

51. The valency of the element having configuration 1s2, 2s22p6, 3s23p2 is [DCE]
(a) +4 (b) +2 (c) +1 (d) –2

52. Diffusion of He gas is [DCE]


(a) 4 times faster than CO2 (b) 4 times faster than SO2
(c) 4 times faster than NO2 (d) 4 times faster than ClO2

53. The hybridisation in PF3 is [DCE]


(a) sp 3 (b) sp 2 (c) dsp 3 (d) d 2 sp 3

[ 43 ]
Periodic Classification of Elements and Chemical Bonding

54. Which is not Paramagnetic? [DCE]


(a) O 2 (b) O 2 + (c) O 2 2– (d) O 2 –

55. In which block 106th element belongs [DCE]


(a) s-block (b) p-block (c) d-block (d) f -block

56. The state of hybridisation in PCl5 is [DCE]


(a) sp 3 d (b) dsp 3 (c) sp 3 (d) sp 3 d 2

57. Whose name is not associated with the development of Periodic Table? [DCE]
(a) Prout’s (b) Newlands (c) Rutherford (d) Loother Meyer

58. Polarizability of halide ions increases in the order [DCE]


(a) F–, I–, Br–, Cl– (b) Cl–, Br–, I–, F– (c) I–, Br–, Cl–, F– (d) F–, Cl–, Br–, I–

59. An element X which occurs in the first short period has an outer electronic structure s2p1. What
are the formula and acid-base character of its oxides? [DCE]
(a) XO3, basic (b) X2O3, basic (c) X2O3, acidic (d) XO2, acidic

60. Three centred bond is present in [DCE]


(a) NH 3 (b) B2 H 6 (c) BCl3 (d) AlCl3

61. Diagonal relationship exist between [DCE]


(a) Li and Mg (b) Na and Mg (c) K and Mg (d) Al and Mg

62. Which element has the highest electro-negativity? [DCE]


(a) F (b) He (c) Ne (d) Na

63. The correct arrangement in increasing order of size for the four isoelectronic species is [DCE]
(a) S2– > Cl > K+ > Ca2+ (b) Ca2+ < K+ < Cl– < S2–
(c) Ca2+ > Cl– > S2– > K+ (d) K+ > Cl– > Ca2+ > S2–

64. A molecule with the highest bond energy is [DCE]


(a) bromine (b) fluorine (c) chlorine (d) iodine

65. N2 is diamagnetic and O2 is paramagnetic. Both the molecules have even number of electrons
(N2 : 14; O2 : 16). It is not true that [DCE]
(a) the energy of the two orbitals x2px and x2py in O2 is the same
(b) there are two unpaired electrons in O2
(c) the bond order in N2 is 3
(d) the bond order in O2 is 3

66. Pick out the electronic configuration of the most electropositive element [DCE]
(a) ns 2 np 3 (b) ns 2 np 0 (c) ns 2 np 1 (d) ns 2 np 4

67. The correct order of increasing oxidising power is [DCE]


(a) F2 < Cl2 < Br2 < I2 (b) F2 < Br2 < Cl2 < I2 (c) Cl2 < Br2 < F2 < I2 (d) I2 < Br2 < Cl2 < F2

[ 44 ]
Periodic Classification of Elements and Chemical Bonding

68. Molecular O2 contains two unpaired electrons. They are [DCE]


(a) * and  (b) * and  (c) * and * (d) * and *

69. The three sets of elements are given below:


(i) Boron, Aluminium, Gallium, Beryllium (ii) Boron, Aluminium, Gallium, Germanium
(iii) Boron, Aluminium, Thallium, Rubidium
Which of the above contains groups of elements whose properties are similar in many respects to
group 13 elements? [DCE]

(a) (ii) (b) (iii) (c) (i) (d) (i) & (iii)

70. An element with atomic number 20 is [DCE]


(a) an alkali metal (b) an alkaline earth metal
(c) a halogen (d) a noble base

71. Ni, Pt, Pd belong to which group of the periodic table? [DCE]
(a) 12th (b) 14th (c) 8th (d) 10th

72. The outermost electronic configuration of the most electronegative element is [DCE]
(a) ns 2 np 3 (b) ns 2 np 4 (c) ns 2 np 5 (d) ns 2 np 6

73. Shape of CO2 is [DCE]

(a) tetrahedral (b) trigonal (c) bent (d) linear

74. Which one of the following belongs to representative group of elements in the Periodic Table?
(a) Lanthanum (b) Argon
(c) Chromium (d) Aluminium [DCE]

75. Which of the following contains both covalent and ionic bonds? [DCE]

(a) CCl4 (b) CaCl2 (c) NH4Cl (d) H 2 O

76. Keeping in view the periodic law, and the periodic table, suggest which of the following elements
should have the maximum electronegative character? [DCE]
(a) Oxygen (b) Nitrogen (c) Fluorine (d) Astatine

77. The electronic configuration of element atomic number 37 is [DCE]


(a) (2, 8) 3s23p63d104s24p65s1 (b) (2, 8) 3s23p63d104s25s64p5
(c) (2, 8) 3s23p64s23d 9 5s14p5 (d) none of these

78. CO2 is isostructural with [DCE]

(a) C2 H 2 (b) SnCl2 (c) NO 2 (d) MgCl2

79. Which of the following is having highest bond length? [DCE]

(a) NO – (b) NO + (c) CN – (d) CN+

[ 45 ]
Periodic Classification of Elements and Chemical Bonding

80. Which of the following are possible resonating structure of N2O? [DCE]

.. .. .. .. .. ..
N = N+ = O
..
:N – N+  O: :N  N+ – O
..
: N
..
= O+ = N
..
.. ..

1 2 3 4
(a) 1 and 2 (b) 1 and 3 (c) 1, 2 and 3 (d) all of these

81. What is the reaction for unusual high B.P. of water. [DCE]
(a) due to the presence H+ and OH– ions in water.
(b) due to dipole-dipole interactions.
(c) due to London forces.
(d) strong London forces.

82. Which of the following statements is wrong? [DCE]


(a) metals are more than nonmetals
(b) there are only few Metalloids
(c) hydrogen can be placed with alkali metals as well as with halogen in periodic table
(d) non metals are more than metals

83. N2 accept e– and convert into N2–, where this e– goes? [DCE]

(a) antibonding  molecular orbital (b) bonding  molecular orbital


(c)  bonding molecular oribital (d)  anti-bonding molecular oribital

84. Lattice energy of an ionic compound depends upon [AIEEE]

(a) Charge on the ion only (b) Size of the ion only
(c) Packing of ions only (d) Charge on the ion and size of the ion

85. In which of the following arrangements the order is NOT according to the property indicated
against it? [AIEEE]

(a) Al3+ < Mg2+ < Na+ < F– : (b) B < C < N < O :
Increasing ionic size Increasing first ionization enthalpy
(c) I < Br < F < Cl : (d) Li < Na < K < Rb :
Increasing electron gain enthalpy Increasing metallic radius
(with negative sign)

86. Of the following sets which one does NOT contain isoelectronic species ? [AIEEE]

(a) PO34 , SO24  , Cl4 (b) CN–, N2, C22  ,

(c) SO23  , CO32  , NO3 (d) BO33 , CO33 , NO3

87. The lanthanide contraction is responsible for the fact that [AIEEE]
(a) Zr and Y have about the same radius (b) Zr and Nb have similar oxidation state
(c) Zr and Hf have about the same radius (d) Zr and Zn have the same oxidation state

[ 46 ]
Periodic Classification of Elements and Chemical Bonding

88. Which one of the following ions has the highest value of ionic radius? [AIEEE]

(a) O 2– (b) B3+ (c) Li+ (d) F –

89. The correct order of bond angles (smallest first) in H2S, NH3, BF3 and SiH4 is [AIEEE]
(a) H2S < NH3 < SiH4 < BF3 (b) NH3 < H2S < SiH4 < BF3
(c) H2S < SiH4 < NH3 < BF3 (d) H2S < NH3 < BF3 < SiH4

90. Which one of the following sets of ions represents the collection of isoelectronic species?
(a) K+, Cl–, Mg2+, Sc3+ (b) Na+, Ca2+, Sc3+, F–
(c) K+, Ca2+, Sc3+, Cl– (d) Na+, Mg2+, Al3+, Cl–
(Atomic nos.: F = 9, Cl = 17, Na = 11, Mg = 12, Al = 13, K = 19, Ca = 20, Sc = 21) [AIEEE]

91. The bond order in NO is 2.5 while that in NO+ is 3. Which of the following statements is true for
these two species? [AIEEE]
(a) Bond length in NO+ is equal to that in NO (b) Bond length in NO is greater than in NO+
(c) Bond length in NO+ is greater than in NO (d) Bond length is unpredictable

92. Of the following outer electronic configurations of atoms, the highest oxidation state is achieved
by which one of them? [AIEEE]
(a) (n –1)d3 ns2 (b) (n –1)d5 ns1 (c) (n –1)d8 ns2 (d) (n –1)d5 ns2

93. The maximum number of 90° angles between bond pair of electrons is observed in
(a) dsp2 hybridization (b) sp3d hybridization
(c) dsp3 hybridization (d) sp3d2 hybridization [AIEEE]

94. According to the Periodic Law of elements, the variation in properties of elements is related to
their
(a) nuclear masses (b) atomic numbers
(c) nuclear neutron-proton number ratios (d) atomic masses [AIEEE]

95. A reduction in atomic size with increase in atomic number is a characteristic of elements of
(a) d-block (b) f -block
(c) radioactive series (d) high atomic masses [AIEEE]

96. Which one of the following groupings represents a collection of isoelectronic species? [AIEEE]

(At. nos.: Cs : 55, Br : 35)


(a) N3–, F–, Na+ (b) Be, Al3+, Cl– (c) Ca2+, Cs+, Br (d) Na+, Ca2+, Mg2+

97. Which one of the following compounds has the smallest bond angle in its molecule? [AIEEE]

(a) OH 2 (b) SH2 (c) NH 3 (d) SO3

98. A square planar complex is formed by hybridisation of which atomic orbitals? [AIEEE]

(a) s, px, py, d y z (b) s, px, py, dx2–y2 (c) s, px, py, dz2 (d) s, px, py, dxy

[ 47 ]
Periodic Classification of Elements and Chemical Bonding

99. Which of the following are arranged in an increasing order of their bond strengths? [AIEEE]
(a) O2– < O2 < O2+ < O22– (b) O22– < O2– < O2 < O2+
(c) O2– < O22– < O2 , O2+ (d) O2+ < O2 < O2– < O22–

100. Which is the correct order of ionic sizes? [AIEEE]


(a) Ce > Sn > Yb > Lu (b) Sn > Ce > Lu > Yb
(c) Lu > Yb > Sn > Ce (d) Sn > Yb > Ce > Lu
(At. nos.: Ce = 58, Sn = 50, Yb = 70 & Lu = 71)

101. In XeF2, XeF4 and XeF6, the number of lone pairs on Xe is respectively [AIEEE]
(a) 2, 3, 1 (b) 1, 2, 3 (c) 4, 1, 2 (d) 3, 2, 1

102. In which of the following species is the underlined carbon having in sp3-hybridisation? [AIEEE]

(a) CH C OOH (b) CH 3 C H 2 OH (c) CH 3 C OCH 3 (d) CH 2  C H  CH 3

103. Which of the following molecules/ions does not contain unpaired electrons? [AIEEE]
2– +
(a) O 2 (b) O 2 (c) B 2 (d) N 2

104. The increasing order of the first ionization enthalpies of the elements B, P, S and F (lowest first is)
(a) B < S < P < F (b) F < S < P < B (c) P < S < B < F (d) B < P < S < F

105. A metal, M forms chlorides in its + 2 and +4 oxidation states. Which of the following statements
about these chlorides is correct? [AIEEE]
(a) MCl2 is more easily hydrolysed than MCl4
(b) MCl2 is more volatile than MCl4
(c) MCl2 is more soluble in anhydrous ethanol than MCl4
(d) MCl2 is more ionic than MCl4

106. Which one of the following sets of ions represents of collection of isoelectronic
species? [AIEEE]
(a) Li+, Na+, Mg2+, Ca2+ (b) K+, Cl–, Ca2+, Sc3+ (c) Ba2+, Sr2+, K+, Ca2+ (d) N3–, O2–, F–, S2–

107. In which of the following molecules/ions are all the bonds not equal? [AIEEE]
(a) BF 4 – (b) SF4 (c) SiF4 (d) XeF 4

108. The decreasing values of bond angles from NH3 (106°) to SbH3 (101°) down group 15 of the
periodic table is due to [AIEEE]
(a) decreasing electronegativity (b) increasing bp – bp repulsion
(c) increasing p-orbital character in sp3 (d) decreasing lp – bp repulsion

109. Following statements regarding the periodic trends of chemical reactivity of the alkali metals and the
halogens are given. Which of these statements gives the correct picture? [AIEEE]
(a) In alkali metals the reactivity increases but in the halogens it decreases with increase in atomic
number down the group
(b) The reactivity decreases in the alkali metals but increases in the halogens with increase in
atomic number down the group
(c) In both the alkali metals and the halogens the chemical reactivity decreases with increase in
atomic number down the group
(d) Chemical reactivity increases with increase in atomic number down the group in both the alkali
metals and halogens
[ 48 ]
Periodic Classification of Elements and Chemical Bonding

110. Which of the following hydrogen bonds is the strongest? [AIEEE]


(a) O – H ... O (b) O – H ....F (c) O – H .... N (d) F – H .... F

111. The charge/size ratio of a cation determines its polarizing power. Which one of the following
sequences represents the increasing order of the polarizing power of the cationic species, K+,
Ca2+, Mg2+, Be2+ ? [AIEEE]
(a) K+ < Ca2+ < Mg2+ < Be2+ (b) Ca2+ < Mg2+ < Be2+ < K+
(c) Mg2+ < Be2+ < K+ < Ca2+ (d) Be2+ < K+ < Ca2+ < Mg2+

112. In which of the following ionization processes, the bond order has increased and the magnetic
behaviour has changed? [AIEEE]
(a) O2  O2+ (b) N2  N2+ (c) C2  C2+ (d) NO  NO+

113. Which one of the following pairs of species have the same bond order? [AIEEE]
(a) CN– and CN+ (b) O2– and CN– (c) NO+ and CN+ (d) CN– and NO+

114. Which of the following constitutes a group of the isoelectronic species? [AIEEE]
+ 2– – – 2– 2–
(a) NO , C2 , CN , N2 (b) CN , N2, O2 , C2
(c) N2, O2–, NO+, CO (d) C22–, O2–, CO, NO

115. Using MO theory predict which of the following species has the shortest bond length? [AIEEE]
(a) O2 (b) O2 (c) O22  (d) O22 

116. In which of the following arrangements, the sequence is not strictly according to the property written
against it? [AIEEE]
(a) HF HCl  HBC  HI: increasing acid strength
(b) NH3  PH3  AsH3< SbH3 : increasing basic strength
(c) B < C < O < N: increasing first ionisation enthalpy
(d) CO2 < SiO2 < SnO2 < PbO2 : increasing oxidising power

117. The set representing the correct order of ionic radius is: [AIEEE]
(a) Na+  Li+  Mg2+ Be2+ (b) Li+  Na+  Mg2+ Be2+
(c) Mg2  Be2+  Li+ Na+ (d) Li+  Be2+  Na+ Mg2+

118. The energy required to break one mole of Cl – Cl bonds in Cl2 is 242 kJ mol–1 . The longest
wavelength of light capable of breaking a single Cl – Cl bond is (c = 3 × 10 8 ms –1 and
NA = 6.02 × 1023 mol–1) [AIEEE]
(a) 700 nm (b) 494 nm (c) 594 nm (d) 640 nm

119. The correct sequence which shows decreasing order of the ionic radii of the
elements is [AIEEE]
(a) Na+  F–  Mg2+  O2–  Al3+ (b) O2–  F–  Na+  Mg2+  Al3+
(c) Al3+  Mg2+  Na+  F–  O2– (d) Na+  Mg2+  Al3+  O2–  F–

[ 49 ]
Periodic Classification of Elements and Chemical Bonding

120. The hybridisation of orbitals of N atom in NO3 , NO 2 and NH 4 are respectively: [AIEEE]

(a) sp, sp2, sp3 (b) sp2, sp, sp3 (c) sp, sp3, sp2 (d) sp2, sp3, sp

121. The structure of IF7 is: [AIEEE]


(a) square pyramid (b) trigonal bipyramid
(c) octahedral (d) pentagonal bipyramid

122. The outer electron configuration of Gd (Atomic No: 64) is: [AIEEE]
(a) 4f3 5d5 6s2 (b) 4f8 5d0 6s2 (c) 4f4 5d4 6s2 (d) 4f7 5d1 6s2

123. The increasing order of the ionic radii of the given isoelectronic species is: [AIEEE]
(a) Cl–, Ca2+, K+, S2– (b) S2–, Cl–, Ca2+, K+ (c) Ca2+, K+, Cl–, S2– (d) K+, S2–, Ca2+, Cl–

124. In which of the following pairs the two species are not isostructural? [AIEEE]

(a) CO32 and NO3 (b) PCl4 and SiCl4 (c) PF5 and BrF5 (d) AlF63 and SF6

125. The molecule having smallest bond angle is [AIEEE]

(a) NCl3 (b) AsCl3 (c) SbCl3 (d) PCl3

126. Which of the following atoms has the highest first ionization energy? [JEE Mains]
(a) Sc (b) Rb (c) Na (d) K

127. The increasing order of atomic radii of the following Group 13 elements is [JEE Advance]
(a) Al < Ga < In < Tl (b) Ga < Al < ln < Tl (c) Al < ln < Ga < Tl (d) Al < Ga < Tl < ln

128. The group having isoelectronic species is [JEE Mains]


(a) O2–, F–, Na+, Mg2+ (b) O–, F–, Na, Mg+ (c) O2–, F–, Na, Mg2+(d) O–, F–, Na+, Mg2+

129. According to molecular orbital theory, which of the following will not be a viable
molecule? [JEE Mains]
2– 2+ + –
(a) H2 (b) He2 (c) He2 (d) H 2

130. Which of the following compounds contain(s) no covalent bonds (s)? [JEE Mains]
KCl, PH3, O2, B2H6, H2SO4
(a) KCl, B2H6 (b) HCl, B2H6, PH3 (c) KCl, H2SO4 (d) KCl

131. Total number of lone pair of electrons in I3– ion is [JEE Mains]
(a) 12 (b) 3 (c) 6 (d) 9

[ 50 ]
Periodic Classification of Elements and Chemical Bonding

CHAPTER TEST
SECTION - I: MULTIPLE CHOICE QUESTIONS (Single Correct)

1. Which of the following represents the correct order of increasing first ionization enthalpy for Ca, Ba,
S, Se and Ar?
(a) Ba < Ca < Se < S < Ar (b) Ca < Ba < S < Se < Ar
(c) Ca < S < Ba < Se < Ar (d) S < Se < Ca < Ba < Ar

2. In which of the following pairs of molecules / ions, both the species are not likely
to exist?

(a) H 22 , He2 (b) H 2 , He22 (c) H 2 , He22 (d) H 2 , He22

3. The first ionisation potential of Na is 5.1 eV. The value of electron gain enthalpy of
Na+ will be:
(a) –10.2 eV (b) + 2.55 eV (c) –2.55 eV (d) –5.1 eV

4. Stability of the species Li 2 , Li 2 and Li 2 increases in the order of:

(a) Li 2  Li2  Li 2 (b) Li 2  Li2  Li 2 (c) Li 2  Li2  Li 2 (d) Li 2  Li 2  Li 2

5. Which of the following arrangements does not represent the correct order of the property stated
against it?
(a) Co3+ < Fe3+ < Cr3+ < Sc3+ : stability in aqueous solution
(b) Sc < Ti < Cr < Mn : number of oxidation states
(c) V2+ < Cr2+ < Mn2+ < Fe2+ : paramagnetic behaviour
(d) Ni2+ < Co2+ < Fe2+ < Mn2+ : ionic size

SECTION - II: MULTIPLE CHOICE QUESTIONS (More than one correct)

6. Which one of the following molecule(s) is (are) expected to exhibit diamagnetic behaviour?
(a) O 2 (b) S 2 (c) C 2 (d) N 2

7. The iso-structural molecules/ions among the following are:


(a) CO32 (b) NO3 (c) ClO3 (d) ClF3

8. The molecules or ions having V-shape are


(a) OF 2 (b) NH 2 (c) I3 (d) OF 2

9. Select the incorrectly matched in the following:


(a) Bridge element: K, Mg
(b) Diagonal relationship: (Li  Mg); (Be  Al) ; (B  Si)
(c) First lanthanoid : La
(d) Typical elements: Na, Mg, Al, S

10. Which of the following pairs of elements have almost similar atomic radii?
(a) Mo, W (b) Zr, Hf (c) Nb, Ta (d) Zn, Cd

[ 51 ]
Periodic Classification of Elements and Chemical Bonding

SECTION - III: INTEGER TYPE QUESTIONS

11. Calculate the effective nuclear charge at the periphery of N-atom.

12. Highest oxidation state of Cl in its oxy acids is.

13. The four pair ionisation energy values of an element are 191, 578, 872 and 5962 Kcal. The number
of valence electrons in the element is.

14. How many of the following makes stable dipositives ions: Na, Mg, K, Ca, Cu, Zn, O, N, Cl?

15. How many of the following are isoelectronic?


Al3+, Mg2+, Cl– , Na+, F–, O2–, S2–, N3–, P3–

16. How many of the following are sp3 hybridized?


NH4+, NH3, H2O, H3O+, CH4, SiH4, CCl4, PCl3, SF4, XeF4, CO2

IE  EA
17. According to mulliken scale electronegativity of an atoms is: XM = and a is
a

18. What is the magnitude of electron gain enthalpy in eV of Na+ if 1E1 of Na is 5.0 eV?

SECTION - IV: MATRIX MATCH

19. Column - I (Species) Column - II (Nature/Shape)


(A) XeF4 (p) Planar
(B) AsF 5 (q) Non-planar
(C) SOCl2 (r) Polar
(D) XeO2F 2 (s) Non-polar

SECTION - V: COMPREHENSION TYPE QUESTIONS

Comprehension: According to ‘Valence-bond theory’, the atomic orbitals (or hybrid orbitals) are thought to
retain their identity even when the atoms are chemically bonded in a molecule and the electrons still occupy
atomic orbitals. However, according to more logical and better developed ‘molecular orbital theory’, the
valence electrons are considered to be associated with all the nuclei in the molecule, i.e., they are spread over
the entire molecule.

20. N2 and O2 are converted into monoanions, N 2 and O2 respectively. Which of the following statements
is wrong?
(a) In N 2 , N-N bond weakens (b) In O2 , O–O bond order increases

(c) In O2 , O–O bond order decreases (d) N 2 becomes paramagnetic

21. Which one of the following should be most stable?


(a) H 2 (b) H + (c) H (d) H–

22. Assuming X-axis as the internuclear axis, molecular orbital(s) containing only one nodal plane is/are
*
(a) σ*2 s (b) σ 2 px (c) π 2 py (d) All the three

[ 52 ]
Periodic Classification of Elements and Chemical Bonding

ANSWERS
CHAPTER ASSIGNMENT
MULTIPLE CHOICE QUESTIONS (Single Correct)

1. (d) 2. (d) 3. (d) 4. (b) 5. (c)


6. (a) 7. (a) 8. (a) 9. (c) 10. (b)
11. (a) 12. (c) 13. (c) 14. (a) 15. (b)

INTEGER TYPE QUESTIONS

16. (8) 17. (4) 18. (8) 19. (5) 20. (6)
21. (7) 22. (4) 23. (7) 24. (4) 25. (0)

MULTIPLE CHOICE QUESTIONS (More one Correct)

26. (a), (b), (c) 27. (c), (d) 28. (a), (b),(c) 29. (a), (b), (c) 30. (a), (c)
31. (b), (c) 32. (a), (c) 33. (a), (b) 34. (a), (b), (c) 35. (b), (d)

MATRIX MATCH TYPE QUESTIONS

36. A – q, r; B – q, r; C – p, s; D – p, s
37. A – p, s; B – q, r; C – q, r; D – q, r

COMPREHENSION TYPE QUESTIONS

38. (b) 39. (d) 40. (b) 41. (b) 42. (c)
43. (c).

PREVIOUS YEAR QUESTIONS

1. (c) 2. (c) 3. (a) 4. (a) 5. (b)


6. (a) 7. (c) 8. (d) 9. (b) 10. (c)
11. (d) 12. (a) 13. (d) 14. (b) 15. (b)
16. (a) 17. (c) 18. (c) 19. (c) 20. (c)
21. (a) 22. (a) 23. (b) 24. (c) 25. (c)
26. (c) 27. (a) 28. (c) 29. (b) 30. (b)
31. (a) 32. (c) 33. (b) 34. (a) 35. (d)
36. (b) 37. (b) 38. (d) 39. (a)
40. A-(p),(q),(r),(t); B-(p),(q),(r),(s),(t); C-(p),(q),(r),(t); D-(p),(q),(r),(s),(t)
41. (6) 42. (4) 43. F, Na 44. (8) 45. (d)
46. (8) 47. (a,b,d) 48. (3) 49. (a) 50. (d)
51. (a) 52. (b) 53. (b) 54. (c) 55. (c)
56. (a) 57. (c) 58. (d) 59. (c) 60. (b)
61. (a) 62. (a) 63. (d) 64. (c) 65. (c)
66. (c) 67. (d) 68. (d) 69. (c) 70. (b)
71. (d) 72. (c) 73. (d) 74. (d) 75. (c)
76. (c) 77. (d) 78. (a) 79. (a) 80. (b)

[ 53 ]
Periodic Classification of Elements and Chemical Bonding

81. (b) 82. (d) 83. (a) 84. (d) 85. (b)
86. (c) 87. (c) 88. (a) 89. (a) 90. (c)
91. (b) 92. (d) 93. (d) 94. (b) 95. (b)
96. (a) 97. (a) 98. (b) 99. (b) 100. (a)
101. (d) 102. (b) 103. (b) 104. (a) 105 .(d)
106. (b) 107. (b) 108. (c) 109. (a) 110. (d)
111. (a) 112. (d) 113. (d) 114. (a) 115. (d)
116. (b) 117. (a) 118. (b) 119. (b) 120. (b)
121. (d) 122. (d) 123. (c) 124. (c) 125. (c)
126. (a) 127. (b) 128. (a) 129. (a) 130. (d)
131. (d)

CHAPTER TEST
1. (a) 2. (a) 3. (d) 4. (d) 5. (c)
6. (c, d) 7. (a, b) 8. (a, b, c) 9. (a, c) 10. (a,b, c)
11. (4) 12. (7) 13. (3) 14. (4) 15. (6)
16. (8) 17. (2) 18. (5)
19. A–p, s; B–q, s; C–q, r; D–q, r 20. (b) 21. (d) 22. (d)

[ 54 ]
Periodic Classification of Elements and Chemical Bonding

HINTS AND SOLUTIONS


MULTIPLE CHOICE QUESTIONS (Single Correct)

1. (d) In isoelectronic species, the radii decreases as atomic number increases.


O2– > F– > Na + > Mg2+ > Al3+
Z 8 9 11 12 13
2. (d) Factual
3. (d) Factual
4. (b) Stable configuration as all the three orbitals are singly occupied (b) and (d) belong to same
group but (b) atomic radius is smaller.
5. (c) In halogens, the order is: Cl > F > Br > 1
6. (a) Due to small size of hydrogen, electron is tightly held and thus high energy is required for its
removal.
I.P. of H2 > I.P. of Cl2
7. (a) S from six covalent bonds with six F atoms, i.e., 12 electrons are involved. According to
VSEPR, it has octahedral configuration.
8. (a) AB3 molecule can be either square pyramidal as IF3 are trigonal bipyramid as PCl3.

- -
9. (c) O O O

- - - -
O O O O O O

2 11
B.O.   1.33J
3 -
O O O
10. (b)
-
O N O N O N
-
O O O
Cl -
Cl - Cl
O O O - O O O
O O O
11. (a) BaCl2 BF3 SiCl4 H2O SF6
Bond angle 180° 120° 109.25° 104.5° 90°
12. (c) S is surrounded by five electron pairs (4 bonded and one lone pair). The geometry is trigonal
bipyramidal. The actual shape is see-saw.
13. (c) O2 (17 electrons). It has one orbital singly occupied.
1
14. (a) Bond length  ; B.O of O 22 , O 2 , O 2 and O22 are3.0, 2.5, 1.5 and 1 respectively..
Bond order
15. (b) Negative charge should be present on more electronegative atom.

INTEGER TYPE QUESTIONS


-
O
16. (8) - Unshared pairs of electrons = 8.
O C O

17. (4) BeCl2, AlCl3 are electron deficient whereas

[ 55 ]
Periodic Classification of Elements and Chemical Bonding

O
H 2SO 4 H O S O H
O
Cl Cl

PCl5 Cl P are hypervalent (more than 8 electrons around the central atom)
Cl
Cl
18. (8) XeF2, N2O, I3 , ICl2 , Br3 , HgCl2, N3 and C2H2 are linear whereas the remaining are bent
molecules. Cl
19. (5) F H F F

O C O B B Xe
(μ  0) F F H H
H
(μ  0)
Cl (μ  0) F F
Cl Cl (μ  0)
20. (6) (μ  0)

Cl P

Cl
Cl
Eacb Cl – P – Cl angle is 90°
total 90° angles =6
 1  1 2
21. (7) Li 2  1, Br2  , N 2  2 , N 2  2
2 2
O 2  2.5, O 22  1.0, O2  15, CO32  1.33
C6H6 = 1.5, O3 = 1.5
22. (4) H
O
H
H
H H
O
O H H O
H
H
O
H

23. (7) π bonds = 7

24. (4) H H 2

H O O H H O O
Cu O S
H O O H H O O
H H

Thus, 4 H2O molecules are directly bonded to Cu.

[ 56 ]
Periodic Classification of Elements and Chemical Bonding

25. (0) F
F F

Br

F F

Due to repulsion by the lone pair on the bond pairs, all the four planar bond angles decrease
from 90° to 84.8°. The axial bond also no longer remain 90° with the planar bonds. In other
words, there are no 90° PBrF bonds in BrF5.
MULTIPLE CHOICE QUESTIONS (More one Correct)

26. (a), (b), (c): Same s-character means same hybridisation CH 4, [Ni(CO)4] and XeO3 have
sp3 hybridisation whereas [Ni(CN)4]2– has dsp2 hybridisation.
27. (c), (d): In C2 and N2 all molecular orbitals are fully filled, i.e., there is no unpaired electron present.
Hence, both of them are diamagnetic.

28. (a), (b), (c): Resonance structures may or may not have identical bonding. Therefore, statements
(a), (b) and (c) are correct.
29. (a), (b), (c): Like CO2, HgCl2 and C 2H3 have sp-hybridisation and hence are isostructural,
i.e., linear.

30. (a), (c): Both CN– and NO+ have 14 electron each and hence are isoelectronic. Therefore, they
have identical bond order.
CH3
H3C CH3
31. (b), (c): (  0)
CH3
H3C C H
(  0)
H C CH2CH3

H3C CH2 C H
(  0)
H3C CH2 C H

32. (a), (c): By Fajan’s rules, small size and large charge on cation increase the polarizing power of the
cation and hence increase covalent character.
-
H O F F
(b) (c) Sn
33. (a), (b) : (a) H N H N
Cl Cl
(d) S
O O
H F F

F F F F
O
(b) Cl F (c) Xe (d)
34. (a), (b), (c): (a) S
- C -
O O
F F F F

35. (b), (d): Resonating structures have slightly different energies and no real existence.

[ 57 ]
Periodic Classification of Elements and Chemical Bonding

MATRIX MATCH TYPE QUESTIONS

36. A – q, r; B – q, r; C – p, s; D – p, s
I3 = sp3d, linear, XeF2 = sp3d linear
SnCl2 = sp2, V-shape, SO2 = sp2, V-shape
37. A – p, s; B – q, r; C – q, r; D – q, r
O22 = Bond order 1.0, diamagnetic
O2 = Bond order 2.5, paramagnetic
N 2 = Bond order 2.5, paramagnetic
N 2 = Bond order 2.5, paramagnetic
COMPREHENSION TYPE QUESTIONS
38. (b) Dipole moment of HBr assuming 100% ionic
= 4.8 × 10–10 × 1.41 × 10–8 cm
= 6.578 × 10–18 esu cm
Actual dipole moment
= 0.78 × 10–15 esu cm
0.78  1018
% ionic character = 100  11.68
6.678 1018
39. (d) Amongst the molecules listed, in NH3, the electronegativity difference between N (3.0) and H
(2.1) is the maximum and hence NH3 has the highest dipole moment.
Cl

40. (b)
Cl

Resultant = x 2  x 2  2 x 2 cos120
= x 2  x 2  2 x 2 ( 1/ 2)  x 2  x
Thus, resultant of o-dichlorobenzene is equal to that of chlorobenzene.

O O O
41. (b) ClO3 ion has three bond pairs and one lone pair on the central Cl atom.
O

 1
42. (b) X for MO 2  [5  0  1  0]  2
2
Thus, type of hybridization = sp
 1
X for NO3  [5  0  0  1]  3
2
Thus, type of hybridisation = sp2
 1
X for NH 4  [5  4  1  0]  4
2
Thus, type of hybridization = sp3
Combining all the result, option (b) is correct.
43. (c) AsF3 has sp3d hybridization. In sp3d hybridisation, it is d z 2a orbital which takes part. Hence,
option (c) is correct.

[ 58 ]

You might also like